cart-icon Товаров: 0 Сумма: 0 руб.
г. Нижний Тагил
ул. Карла Маркса, 44
8 (902) 500-55-04

Тест по физике прямолинейное равноускоренное движение: Тест Прямолинейное равноускоренное движение (9 класс) по физике

Содержание

Тест Прямолинейное равноускоренное движение (9 класс) по физике

Сложность: знаток.Последний раз тест пройден более 24 часов назад.

  1. Вопрос 1 из 10

    Какое(-ие) утверждение(-я) верно(-ы): а)равноускоренное движение является неравномерным движением; б)равноускоренное движение является равномерным движением ?

    • Правильный ответ
    • Неправильный ответ
    • Вы и еще 61% ответили правильно
    • 61% ответили правильно на этот вопрос

    В вопросе ошибка?

    Следующий вопросПодсказка 50/50Ответить
  2. Вопрос 2 из 10

    В каких единицах измеряется ускорение в СИ?

    • Правильный ответ
    • Неправильный ответ
    • Вы и еще 75% ответили правильно
    • 75% ответили правильно на этот вопрос

    В вопросе ошибка?

    Подсказка 50/50Ответить
  3. Вопрос 3 из 10

    Какая физическая величина относится к векторным величинам?

    • Правильный ответ
    • Неправильный ответ
    • Вы и еще 73% ответили правильно
    • 73% ответили правильно на этот вопрос

    В вопросе ошибка?

    Подсказка 50/50Ответить
  4. Вопрос 4 из 10

    Какое(-ие) утверждение(-я) верно(-ы): а)если направление ускорения совпадает с направлением начальной скорости, то модуль скорости увеличивается; б)если направление ускорения противоположно направле­нию начальной скорости, то модуль скорости уменьшается ?

    • Правильный ответ
    • Неправильный ответ
    • Вы и еще 63% ответили правильно
    • 63% ответили правильно на этот вопрос

    В вопросе ошибка?

    Подсказка 50/50Ответить
  5. Вопрос 5 из 10

    Мотоциклист начинает движение из состояния покоя. Через 30 с. он достигает скорости 54 км/ч. С каким ускорением происходит движение?

    • Правильный ответ
    • Неправильный ответ
    • Вы и еще 58% ответили правильно
    • 58% ответили правильно на этот вопрос

    В вопросе ошибка?

    Подсказка 50/50Ответить
  6. Вопрос 6 из 10

    Санки съехали с одной горки и въехали на другую. Во время подъема на горку скорость санок, двигавшихся прямолинейно и равноускоренно, за 4 с изменилась от 12 м/с до 2 м/с. При этом ускорение равно

    • Правильный ответ
    • Неправильный ответ
    • Вы и еще 61% ответили правильно
    • 61% ответили правильно на этот вопрос

    В вопросе ошибка?

    Подсказка 50/50Ответить
  7. Вопрос 7 из 10

    Во время подъема в гору скорость велосипедиста, двигающегося прямолинейно и равноускоренно, изменилась за 8 с от 5 м/с до 10,8 км/ч. При этом модуль ускорения велосипедиста был равен

    • Правильный ответ
    • Неправильный ответ
    • Вы ответили лучше 55% участников
    • 45% ответили правильно на этот вопрос

    В вопросе ошибка?

    Подсказка 50/50Ответить
  8. Вопрос 8 из 10

    Велосипедист съезжает с горки, двигаясь прямолинейно и равноускоренно. За время спуска скорость велосипедиста увеличилась на 10 м/с. Ускорение велосипедиста 0,5 м/с2. Сколько времени длится спуск?

    • Правильный ответ
    • Неправильный ответ
    • Вы ответили лучше 51% участников
    • 49% ответили правильно на этот вопрос

    В вопросе ошибка?

    Подсказка 50/50Ответить
  9. Вопрос 9 из 10

    Ускорение велосипедиста на одном из спусков трассы равно 1,2 м/с2. На этом спуске его скорость увеличилась на 18 м/с. Велосипедист заканчивает свой спуск после его начала через

    • Правильный ответ
    • Неправильный ответ
    • Вы и еще 59% ответили правильно
    • 59% ответили правильно на этот вопрос

    В вопросе ошибка?

    Подсказка 50/50Ответить
  10. Вопрос 10 из 10

    Вагон наехал на тормозной башмак при скорости 4,5 км/ч. Через 3 с вагон остановился. Определите тормозной путь

    • Правильный ответ
    • Неправильный ответ
    • Вы ответили лучше 56% участников
    • 44% ответили правильно на этот вопрос

    В вопросе ошибка?

    Подсказка 50/50Ответить

Доска почёта

Чтобы попасть сюда — пройдите тест.

ТОП-5 тестовкоторые проходят вместе с этим

Изучение ряда тем по физике имеет огромное значение для того, чтобы школьник мог успешно знакомиться с основами этой науки, а потому тест «Прямолинейное равноускоренное движение» (9 класс) может стать особенно важен для всех, кто хочет хорошо или отлично успевать по предмету. Он содержит в себе десять вопросов разной степени сложности; задания с выбором правильного ответа требуют знания теории («равноускоренное движение», «ускорения», «единицы СИ») и умения описывать физические опыты и решать небольшие по объему задачи (например, на поиск ускорения).

Тесты по физике «Равноускоренное движение» пригодятся не только девятиклассникам; они могут быть очень удобны и для старшеклассников, которые повторяют изученное перед экзаменом ЕГЭ. Пригодятся они также родителям или учителям, которым требуется провести урок контроля знаний.

Рейтинг теста

Средняя оценка: 3.5. Всего получено оценок: 600.

А какую оценку получите вы? Чтобы узнать — пройдите тест.

Тест по Физике «Равноускоренное прямолинейное движение» 9 класс

ДЕПАРТАМЕНТ ОБРАЗОВАНИЯ ГОРОДА МОСКВЫ

Государственное бюджетное образовательное учреждение

среднего профессионального образования города Москвы

Колледж сферы услуг № 32

УТВЕРЖДАЮ

Зам. директора по УМР

____________ Л. А. Васильева

«____»___________ 2013г

Комплект заданий для самообследования

предмету Физика

260807.01 «Повар, кондитер»

Рассмотрено

предметной (цикловой) комиссией

______________________________

Протокол №___________________

от «___» ______________20 _____г.

Председатель предметной (цикловой) комиссии

___________________/ ______________________

подпись ФИО

Москва

2013

тест по физике. 9 класс

вариант 1

  1. В каком из следующих случаев движение тела можно рассматривать как движение материальной точки?

А. Движение автомобиля из одного города в другой.

Б. Движение конькобежца, выполняющего программу фигурного катания.

В. Движение поезда на мосту.

Г. Вращение детали, обрабатываемой на станке.

2. При равноускоренном движении скорость тела за 6 с изменилась от 6 м/с до 18 м/с. Определите ускорение тела.

А. 4 м/с2; Б. 2 м/с2; В. -2 м/с2; Г. 3 м/с2.

3. Земля притягивает к себе тело массой 1,5 кг с силой:

А. 1,5 Н; Б. 15 Н; В. 0,15 Н; Г. 150 Н.

5. Какая из приведенных формул выражает закон всемирного тяготения?

А. ; Б. ; В. ; Г. .

6. Тело массой 2 кг движется со скоростью 5 м/с. Определите импульс тела. Как он направлен?

А. 5 кг∙м/с, импульс не имеет направления.

Б. 10 кг∙м/с, в сторону, противоположную направлению скорости тела.

В. 10 кг∙м/с, совпадает с направлением скорости тела.

Г. Среди ответов нет правильного

7. Что совершает колебания в механической волне?

А. Скорость; Б. Частицы среды; В. Энергия; Г. Ускорение.

8. Электрический ток создает вокруг себя:

А. Электрическое поле; Б. Магнитное поле;

9.При последовательном соединении проводников общее напряжение на участке цепи…

А. Такое же, как и на отдельных проводниках.

Б. Равно сумме напряжений на отдельных проводниках.

В. Равно сумме обратных величин напряжений на от¬дельных проводниках.

Г. Меньше, чем напряжение на отдельных проводниках.

Д. Больше, чем напряжение на отдельных проводниках.

10. По какой формуле вычисляют мощность электри¬ческого тока?

А. P=U/It Б. P=UI В. P=I/U

Г. P=UR Д. P=U/R

тест по физике. 9 класс

вариант 2

1. В каком из следующих случаев движение тела нельзя рассматривать как движение материальной точки?

А. Движение автомобиля из одного города в другой.

Б. Движение конькобежца, выполняющего программу фигурного катания.

В. Движение поезда на мосту.

Г. Вращение детали, обрабатываемой на станке.

2. При равноускоренном движении скорость тела за 12 с изменилась от 6 м/с до 18 м/с. Определите ускорение тела.

А. 4 м/с2; Б. 2 м/с2; В. -2 м/с2; Г. 1 м/с2.

3. Земля притягивает к себе тело массой 3 кг с силой:

А. 0,03 Н; Б. 30 Н; В. 0,3 Н; Г. 300 Н.

4. . Мощность электрического тока на участке цепи равна произведению…

А. Напряжения на концах этого участка на время про¬хождения тока.

Б. Напряжения на кон¬цах этого участка на силу тока и на время.

В. Силы тока на нап¬ряжение на концах этого участка.

Г. Сопротивления этого участка на силу тока и на время.

Д. Сопротивления этого участка на напря¬жение на его концах и на время.

5. Тело массой 4 кг движется со скоростью 5 м/с. Определите импульс тела. Как он направлен?

А. 5 кг∙м/с, импульс не имеет направления.

Б. 20 кг∙м/с, в сторону, противоположную направлению скорости тела.

В. 20 кг∙м/с, совпадает с направлением скорости тела.

Г. Среди ответов нет правильного.

6. Тело на пружине совершает колебания. Под действием какой силы совершаются колебания, назовите ее.

А. — сила упругости; Б. — сила тяжести;

В. — сила упругости; В. — сила трения.

7. Что совершает колебания в механической волне?

А. Скорость; Б. Частицы среды; В. Энергия; Г. Ускорение.

8. Заряженная частица создает вокруг себя:

А. Электрическое поле; Б. Магнитное поле;

9.Работа электрического тока на участке цепи равна произведению…

А. Напряжения на концах этого участка на время про¬хождения тока.

Б. Сопротивления этого участка на напря¬жение на его концах и на время.

В. Сопротивления этого участка на силу тока и на время.

Г. Силы тока на напряже¬ние на концах этого участка.

Д. Напряжения на концах этого участка на силу тока и на время.

10. По какой формуле вычисляют работу электриче¬ского тока?

А. A=UI Б. A=UI/t В. A=U/R

Г. А=IUt Д. A=UIR

Тест по теме «Прямолинейное равноускоренное движение. Ускорение»

Тест 9 класс. Прямолинейное равноускоренное движение. Ускорение.

 

Задание1

Какое из утверждений является верным?

1) равноускоренное движение — это неравномерное движение

2) равноускоренное движение — это равномерное движение

 

Задание 2

Какая из приведенных ниже формул соответствует определению ускорения?

 

1)              2)                  3)

4) Среди ответов нет правильного

 

Задание 3

В каких единицах измеряется ускорение в системе СИ

1) м/с           2) м/с2          3) мм/с2       4) м2

 

Задание 4

Какая физическая величина относится к векторным величинам?

1) координата           2) путь          3) ускорение         4) траектория

 

Задание 5

Какое (-ие) утверждение (-я) является (-ются) верным (-и)?

     А) если направление вектора ускорения совпадает с направлением начальной скорости тела, то скорость тела увеличивается;

     Б) если направление вектора ускорения противоположно направлению начальной скорости тела, то скорость тела уменьшается

1) ни А, ни Б         2) только А         3) и А, и Б       4) только Б

 

Задание 6

Автомобиль начинает движение из состояния покоя. Через 50 с он достигает скорости 72 км/ч. С каким ускорением происходит движение?

1) -0,4 м/с2          2) 0,4 м/с2           3) 72 м/с2        4) 0,8 м/с2

 

Задание 7

Санки съехали с одной горы и въехали на другую. Во время подъема на гору скорость санок, двигавшихся прямолинейно и равноускорено, за 5 с изменилась от 10 м/с до 2 м/с. Чему равно ускорение санок?

 

1) 1,6 м/с2            2) 8 м/с2        3) -8 м/с2            4) — 1,6 м/с2

Задание 8

Во время подъёма в гору скорость велосипедиста, двигающегося прямолинейно и равноускорено, изменилась за 10 с от 7 м/с до 10,8 км/ч. При этом модуль ускорения велосипедиста был равен

1) -0,25 м/с2           2) 0,25 м/с2         3) -0,5 м/с2          4) 0,5 м/с2

Задание 9

Велосипедист съезжает с горки, двигаясь прямолинейно и равноускоренно. За время спуска его скорость увеличилась на 13 м/с. Ускорение велосипедиста 0,25 м/с

2. Сколько времени длится спуск?

1) 0,52 с        2) 52 с         3) 20 с            4) 0,25 с

Задание 10

Ускорение санок на горке равно 1,6 м/с2. На этом спуске их скорость увеличилась на 36 км/ч. Санки заканчивают свой спуск после его начала через

1) 36 с         2) 625 с       3) 22,5 с            4) 6,25 с

 

Ответы:

1) 1

2) 2

3) 2

4) 3

4) 3

6) 2

7) 4

8) 3

9) 2

10) 4


 

ЕГЭ по физике, подготовка к ЕГЭ по физике 2021 в Москве, задачи, оценки, сколько длится экзамен — Учёба.ру

11Равномерное прямолинейное движение, равноускоренное прямолинейное движение, движение по окружности
21Законы Ньютона, закон всемирного тяготения, закон Гука, сила трения
31Закон сохранения импульса, кинетическая и потенциальные энергии, работа и мощность силы, закон сохранения механической энергии
41Условие равновесия твердого тела, закон Паскаля, сила Архимеда, математический и пружинный маятники, механические волны, звук
52Механика
62Механика
72Механика
81Связь между давлением и средней кинетической энергией, абсолютная температура, связь температуры со средней кинетической энергией, уравнение Менделеева—Клапейрона, изопроцессы
91Работа в термодинамике, первый закон термодинамики, КПД тепловой машины
101Относительная влажность воздуха, количество теплоты
112МКТ, термодинамика
122МКТ, термодинамика
131Принцип суперпозиции электрических полей, магнитное поле проводника с током, сила Ампера, сила Лоренца, правило Ленца
141Закон сохранения электрического заряда, закон Кулона, конденсатор, сила тока, закон Ома для участка цепи, последовательное и параллельное соединение проводников, работа и мощность тока, закон Джоуля — Ленца
151Поток вектора магнитной индукции, закон электромагнитной индукции Фарадея, индуктивность, энергия магнитного поля катушки с током, колебательный контур, законы отражения и преломления света, ход лучей в линзе
162Электродинамика
172Электродинамика
182Электродинамика и основы специальной теории относительности
191Планетарная модель атома. Нуклонная модель ядра. Ядерные реакции
201Фотоны, линейчатые спектры, закон радиоактивного распада
212Квантовая физика
221Механика — квантовая физика
231Механика — квантовая физика
242Элементы астрофизики: Солнечная система, звезды, галактики

тест 2 прямолинейное неравномерное движение вариант 1 ответы

тест 2 прямолинейное неравномерное движение вариант 1 ответы

Тест на тему “Неравномерное движение”. Сложность: знаток. Перед прохождением теста рекомендуем прочитать: Неравномерное движение. Загрузка. Рейтинг теста. А какую оценку получите вы? Чтобы узнать — пройдите тест. Новые тесты. Будь в числе первых на доске почета. Предметы. Алгебра. Уроки. Тесты по алгебре.

4. Неравномерное прямолинейное движение. Равноускоренное прямолинейное движение тел. №51. Мотоциклист за первые 2 ч проехал 90 км, а следующие 3 ч двигался со скоростью 50 км/ч. Какова средняя скорость мотоциклиста на всем пути? №52.  Определите характер этого движения. Найдите начальную скорость и ускорение, напишите уравнение зависимости проекции скорости от времени. Что происходит с движущимся телом в момент времени, с. №64. На рисунке 19 приведены графики зависимости vx (t) для двух тел. Определите по каждому графику характер движения тел, найдите проекции начальных скоростей, определите модуль и направление векторов начальной скорости.

Равномерное прямолинейное движение. Тест позволяет проверить умение читать графики прямолинейного равномерного движения. Записывать уравнение движения и решать систему уравнений. Инструкция к тесту. Привыполнении теста будьте внимательны. Ответ записывайте в СИ, если нет специальной оговорки. Удачи!

На этой странице вы рассмотрите Ответы к тестам по физике 9 класс 456456 (Часть 1) из предмета Физика, в предложенной теме также освящены и другие вопросы по Физика. Если у вас появились новые вопросы, спрашивайте в комментариях. Перемещение Вариант 1 1. Механическое движение —- это изменение положения тела в пространстве а) под действием других тел с течением времени б) относительно других тел с течением времени 2. Материальной точкой можно считать самолет, если а) определяют среднюю скорость движения самолета при полете б) вычисляют силу сопротивления воздуха, которая действует на самолет 3.

K какому виду движения — равномерному или неравномерному — относится прямолинейное равноускоренное движение? Что понимают под мгновенной скоростью неравномерного движения? Дайте определение ускорения равноускоренного движения. Какова единица ускорения? Что такое равноускоренное движение? Что показывает модуль вектора ускорения? При каком условии модуль вектора скорости движущегося тела увеличивается; уменьшается? • Вопрос 1 Для прямолинейного равноускоренного движения характерно изменение скорости, поэтому его относят к неравномерному движению. •

Тестирование, бесплатные тесты с ответами и проверка знаний. Какое из ниже перечисленных тел движется равномерно и прямолинейно? Физика. Перемещение при прямолинейном равномерном движении. 9 класс.  Перемещение при прямолинейном равномерном движении. 9 класс. Пройдите тест, узнайте свой уровень и посмотрите правильные ответы! Категория: Физика. Уровень: 9 класс. Какое из ниже перечисленных тел движется равномерно и прямолинейно? экскурсионный автобус. ребенок на качелях.

Тест «Прямолинейное неравномерное движение (вариант 2)» 9 класс. Оценка 4.9 (более 1000 оценок). cloud_queue. Карточки-задания +1. docx. физика.  6. По уравнениям движения определите начальную координату тела и проекции векторов начальной скорости и ускорения: а) x = 10t + 10t2, x0 = _, v0x = _, ax = _, б) x = 4t – 2t2, x0 = _, v0x = _, ax = _. 7. На рисунке показаны положения двух человек в момент времени t = 0, а также их начальные скорости и ускорения. Запишите уравнения движения. x1 = _, x2 = _ .

Тест для 10 класса по физике по теме: «Способы описания движения. Равномерное прямолинейное движение. Сложение скоростей» на два варианта, готовые к двухсторонней печати, + ответы для быстрой проверки. Тест составлен в программе Microsoft Office Word по сборнику Физика, 10 класс, Тесты, часть 1, Сычёв Ю.Н., 2012.

1) равномерное движение; 2) неравномерное движение; 7. Физическая величина, равная отношению перемещения материальной точки к физически малому промежутку времени, в течение которого произошло это перемещение, называется. 1) средней скоростью неравномерного движения материальной точки; 2) мгновенной скоростью материальной точки; 3) скоростью равномерного движения материальной точки. 8. Направление ускорения всегда совпадает с  12.Два поезда движутся навстречу друг другу по прямолинейному участку пути. Один из них движется ускоренно, второй замедленно. Их ускорения направлены

Равномерное прямолинейное движение — Тесты для самоконтроля.  Равномерное прямолинейное движение — Тесты для самоконтроля. Вариант 1. 1. Двигаясь равномерно, велосипедист проезжает 40 м за 4 с. Какой путь он проедет при движении с той же скоростью за 20 с? А. 30 м. Б. 50 м. В. 200 м. 2. На рисунке 1 приведен график движения мотоциклиста. Определите по графику путь, пройденный мотоциклистом в промежуток времени от 2 до 4 с. А. 6 м. Б. 2 м. В. 10 м. 3. На рисунке 2 представлены графики движения трех тел. Какой из этих графиков соответствует движению с большей скоростью? А. 1. Б. 2. В. 3. 4. По графику движения, представленному на рисунке 3, определите скор

Тест с. 4-7. Ответы с. 8. Критерии оценивания с. 9. Список литературы с. 10.  Тест. Механическое движение. Часть А. 1.Линия, по которой движется тело… А. Траектория. Б. Путь.  В. Тело находится в покое или движется неравномерно прямолинейно. Г. Тело находится в покое. 13. Автобус, двигаясь со скоростью 54 км/ч, прошёл путь 45 м за_с.

Равномерное прямолинейное движение — Тесты для самоконтроля.  Равномерное прямолинейное движение — Тесты для самоконтроля. Вариант 1. 1. Двигаясь равномерно, велосипедист проезжает 40 м за 4 с. Какой путь он проедет при движении с той же скоростью за 20 с? А. 30 м. Б. 50 м. В. 200 м. 2. На рисунке 1 приведен график движения мотоциклиста. Определите по графику путь, пройденный мотоциклистом в промежуток времени от 2 до 4 с. А. 6 м. Б. 2 м. В. 10 м. 3. На рисунке 2 представлены графики движения трех тел. Какой из этих графиков соответствует движению с большей скоростью? А. 1. Б. 2. В. 3. 4. По графику движения, представленному на рисунке 3, определите скор

Тесты по физике по теме — Механическое движение с ответами. Правильные ответы обозначены +. 1. Механическим движением тела называют? А. изменение положения тела на плоскости. Б. изменение формы тела с течением времени. В. изменение положения тела в пространстве с течением времени +. 2. Движения бывают? А. поступательные +. Б. переменные.  6. Какие виды неравномерных движений существуют? А. равнозамедленные +. Б. ускоренные.  В. равномерное прямолинейное движение +. Г. движение по наклонной плоскости. 9. Масса тела выражается формулой? А. m=F:a +.

Алгоритм решения ЗАДАЧИ на Прямолинейное равномерное движение. Задачи, описывающие движение, содержат два типа величин: векторные (имеющие направление) и скалярные (выражающиеся только числом). К векторным величинам при описании равномерного прямолинейного движения относятся скорость и перемещение. Для перехода от векторов к скалярам выбирают координатную ось и находят проекции векторов на эту ось, руководствуясь следующим правилом: если вектор сонаправлен с осью, то его проекция положительна, если противоположно направлен — отрицательна.

Прямолинейное равномерное движение-1. Тест (9 класс) | Кинематика. 17.03.2013, 22:14:29. Прямолинейное равномерное движение Вариант №01. За верное выполнение каждого задания — 1 балл. «5» (отлично) — 9-10 баллов; «4» (хорошо) — 7-8 баллов; «3» (удовлетворительно) — 5-6 баллов; «2» (неудовлетворительно) — 0-4 баллов; 1. 0/1. Велосипедист, двигаясь равномерно, проезжает 20 м за 2 с. Определите, какой путь он проедет при движении с той же скоростью за 10 с. А) 60 м. Б) 100 м. В) 150 м. Ответ: 2. 0/1. Определите по графику зависимости пути от времени путь, п

Получи ответ на свой вопрос: 7. На рисунке показаны положения двух человек в момент времени t = 0, а также их начальные скорости и ускорения. Запишите уравнения движения. Тест.  Запишите уравнения движения. Тест «Прямолинейное неравномерное движение (вариант 3)» 9 класс x1 = _, x2 = _ . 8. Пользуясь условием предыдущего вопроса, постройте графики проекций скоростей двух тел и найдите проекции скоростей тел в момент времени t = 1 с. v1x = _ , v2x = _ . Ответ. Ответ дан cvvkl. № 7. x₀₁ = — 2 м x₀₂ = 2 м. v₀₁x = 4 м/с v₀₂x = — 4 м/с. a₁x = — 2 м/с² a₂x = 1 м/с². x₁ = -2 + 4*t — t². x₂ = 2 — 4*t + t² / 2. № 8. v₁x = 4 — 2*t v₁x(1) = 4 — 2*1 = 2 м/с. v₂x = — 4 + t v₂x = -4 + 1 = — 3 м/с.

Урок №4. Тема: «Равномерное прямолинейное движение Решение задач» Равномерное прямолинейное движение — ТЕСТ Вариант I Часть 1 К каждому из заданий 1 – 8 даны 4 варианта ответа, из которых только один правильный. Рассмотрим два вида движения тел: А) троллейбус движется по прямой улице. К каждой следующей остановке он прибывает через равные интервалы времени и через равные интервалы отбывает от них Б) автомобиль движется по дороге и проходит за любые равные промежутки времени одинаковые расстояния В каком случае движение тела является равномерным? 1) только в А 2) только в Б 3) в А и в Б 4) ни в

В комплект входят два варианта теста по теме «Перемещение. Скорость. Равномерное прямолинейное движение» (дидактические материалы А.Е.Марон). Тесты с выбором ответа созданы на основе шаблона А.А.Баженова и позволяют быстро провести оценку качеств  В комплект входят два варианта теста по теме «Перемещение. Скорость. Равномерное прямолинейное движение» (дидактические материалы А.Е.Марон). Тесты с выбором ответа созданы на основе шаблона А.А.Баженова и позволяют быстро провести оценку качества знаний обучающихся. Цель: проверка качества знаний обучающихся по теме «Перемещение. Скорость. Равномерное прямолинейное движение». ©. Коломина Наталья Николаевна.

II вариант. 1. Скорость тела, движущегося по окружности постоянного радиуса, увеличилась в два раза. Центростремительное ускорение: а) увеличилось в 4 раза + б) увеличилось в 2 раза в) не изменилось. 2. Поезд отправляется.  5. Выберите неправильное утверждение: а) при прямолинейном движении с постоянным ускорением скорость может увеличиваться, а может и уменьшаться б) выбор системы отсчёта зависит от условий данной задачи в) направление ускорения определяет направление движения +.

В комплект входят два варианта теста по теме «Перемещение. Скорость. Равномерное прямолинейное движение» (дидактические материалы А.Е.Марон). Тесты с выбором ответа созданы на основе шаблона А.А.Баженова и позволяют быстро провести оценку качеств  В комплект входят два варианта теста по теме «Перемещение. Скорость. Равномерное прямолинейное движение» (дидактические материалы А.Е.Марон). Тесты с выбором ответа созданы на основе шаблона А.А.Баженова и позволяют быстро провести оценку качества знаний обучающихся. Цель: проверка качества знаний обучающихся по теме «Перемещение. Скорость. Равномерное прямолинейное движение». ©. Коломина Наталья Николаевна.

7. Равномерное и неравномерное прямолинейное движение. Сборник задач по физике, Лукашик В.И. 95. В движущемся вагоне пассажирского поезда на столе лежит книга. В покое или движении находится книга относительно: а) стола; б) рельсов; в) пола вагона; г) телеграфных столбов?  99. Какие части велосипеда при прямолинейном движении описывают прямолинейные траектории относительно дороги, а какие — криволинейные? Прямолинейную траекторию описывает рама, криволинейную траекторию описывает точка на ободе. 100. После стыковки космический корабль и орбитальная станция двигались некоторое время совместно.

II вариант. 1. Скорость тела, движущегося по окружности постоянного радиуса, увеличилась в два раза. Центростремительное ускорение: а) увеличилось в 4 раза + б) увеличилось в 2 раза в) не изменилось. 2. Поезд отправляется.  5. Выберите неправильное утверждение: а) при прямолинейном движении с постоянным ускорением скорость может увеличиваться, а может и уменьшаться б) выбор системы отсчёта зависит от условий данной задачи в) направление ускорения определяет направление движения +.

Главная » Тесты » Прямолинейное равномерное движение. Прямолинейное равномерное движение. Тесты по физике 9 класс. В тесте разрешено выбирать только один ответ. Вопросов в тесте: 10. Поиск. Форма входа. Электронный журнал. Вход в сетевой город. Олимпиады и конкурсы.

Вы находитесь на странице вопроса «прямолинейное неравномерное движение тест 2 вариант 3», категории «физика». Данный вопрос относится к разделу «5-9» классов. Здесь вы сможете получить ответ, а также обсудить вопрос с посетителями сайта. Автоматический умный поиск поможет найти похожие вопросы в категории «физика». Если ваш вопрос отличается или ответы не подходят, вы можете задать новый вопрос, воспользовавшись кнопкой в верхней части сайта. Популярное. Antonow00 / 09 нояб. 2016 г., 7:48:11. так тогда отгадайте решение но я знаю ответ проверю вас вот п

1 КИНЕМАТИКА Вариант 1 1. При равномерном движении пешеход проходит за 10 с путь 15 м. Какой путь он пройдет при движений с той же скоростью за 2 с? А. 3 м. Б. 30 м. В. 1,5 м. Г. 7,5 м. 2. На рисунке 1 представлен график зависимости пути, пройденного велосипедистом, от времени. Определите по этому графику путь, пройденный велосипедистом за интервал времени от t 1 =1с до t 2 =3 с. А. 9 м. Б. 6 м. В. 3 м. Г. 12 м. 3. По графику, представленному на рисунке 1, определите скорость движения велосипедиста в момент времени t= 2 с. А. 2 м/с. Б. 6 м/с. В. 3 м/с.  Как изменится центростремительное ускорение тела при увеличении скорости в два раза, если радиус окружности останется неизменным?

Онлайн тест по физике в 9 классе «Прямолинейное равноускоренное движение». 19 вопросов разной формы ответа без ограничения по времени. Перед тестированием рекомендуется прочитать конспекты по проверяемой теме. Результат тестирования оценивается по пятибалльной системе. Чтобы получить оценку «5» необходимо ответить правильно более 80% вопросов. Результат тестирования можно отправить себе на электронную почту (ввод данных в конце теста). Физика 9 класс. Онлайн-тест: Прямолинейное равноускоренное движение. 5%.

• Работа № 2 Прямолинейное неравномерное движение. Движение по окружности. • Работа № 3 Движение тела под действием силы тяжести. • Работа № 4 Динамика.  Тесты Кинематика Вариант 1. 1. Как называется раздел механики изучающий движение тел без выяснения причин этого движения? 1) статика; 2) кинематика; 3) динамика

ТЕСТЫ для САМОКОНТРОЛЯ

ТЕСТЫ для САМОКОНТРОЛЯ
ТСК-9.1.1 — Система отсчёта.Перемещение
ТСК-9.1.2 — Определение координаты тела
ТСК-9.1.3 — Прямолинейное равномерное движение
ТСК-9.1.5 — Прямолинейное равноускоренное движение
ТСК-9.1.6 — Относительность движения
ТСК-9.1.7 — Первый закон Ньютона
ТСК-9.1.8 — Второй закон Ньютона
ТСК-9.1.9 — Третий закон Ньютона
ТСК-9.1.10 — Закон всемирного тяготения
ТСК-9.1.11 — Свободное падение
ТСК-9.1.12 — Криволинейное движение
ТСК-9.1.13 — Искусственные спутники Земли
ТСК-9.1.14 — Импульс тела.Закон сохранения импульса
ТСК-9.1.15 — Закон сохранения энергии
ТСК-9.2.16 — Колебательное движение
ТСК-9.2.17 — Гармонические колебания
ТСК-9.2.18 — Затухающие колебания.Резонанс
ТСК-9.2.19 — Механические волны
ТСК-9.2.20 — Звуковые колебания
ТСК-9.3.21 (повторение) — Магнитное поле
ТСК-9.3.21 — Магнитное поле. Индукция. Магнитный поток
ТСК-9.3.22 — Направление магнитных линий. Правило левой руки
ТСК-9.3.23-24 — Явление электромагнитной индукции
ТСК-9.3.25,26,27 — Переменный ток
ТСК-9.3.28 — Электромагнитное поле
ТСК-9.3.29 — Электромагнитные волны
ТСК-9.3.30 — Получение электромагнитных колебаний
ТСК-9.3.31 — Электромагнитная природа света
ТСК-9.3.32 — Преломление света
ТСК-9.3.33,34— Дисперсия света.Спектры
ТСК-9.4.35,37 — Опыт Резерфорда. Радиоактивные превращения. Методы исследования частиц
ТСК-9.4.36 — Радиоактивность
ТСК-9.4.38 — Состав атомного ядра. Изотопы
ТСК-9.4.39 — Распады.Ядерные силы.Ядерные реакции
ТСК-9.4.40 — Деление ядер урана.Ядерный реактор.Термоядерные реакции

ОТВЕТЫ к ТЕСТАМ для САМОКОНТРОЛЯ


Порядок проведения вступительного испытания по физике (ВШТЭ)

На все направления подготовки в Высшей школе технологии и энергетики экзамен проводится очно с личным присутствием в аудитории, в случае, если это не противоречит постановлениям губернатора Санкт-Петербурга, издаваемым в соответствии с Указом Президента РФ от 11 мая 2020 г. №316 «Об определении порядка продления действия мер по обеспечению санитарно-эпидемиологического благополучия населения в субъектах Российской Федерации в связи с распространением новой коронавирусной инфекции (СOVID-19)», исходя из санитарно-эпидемиологической обстановки и особенностей распространения новой коронавирусной инфекции (СOVID-19).

Вступительный экзамен по физике проводится в виде письменной работы.

1. Продолжительность экзамена – 3 часа (180 минут).
2. Работа выполняется синей или черной пастой. Записи, сделанные карандашом, не проверяются.
3.Разрешается использовать непрограммируемый калькулятор.
4. Все необходимые для решения справочные данные и значения физических постоянных приводятся в условиях задач.
5. Решение задач желательно сопровождать краткими пояснениями.
6. Ответы выписываются дважды: после решения каждой задачи и в сводной таблице, приводимой на четвертой странице чистого двойного листа.
7. Каждое задание содержит 12 задач. №1-6 – задачи-тесты, к каждой из них дается несколько вариантов ответа, из которых верен только один. Каждая задача оценивается в 5 баллов. №7-10 – задачи первого класса сложности. Каждая задача оценивается в 10 баллов. №11, 12 – задачи второго класса сложности, одна задача оценивается в 15 баллов.
8. Максимальная сумма баллов при полном правильном решении равна 100 баллам (6х5+4х10+2х15=100)
9. При проведении экзамена с применением дистанционных технологий абитуриент не позднее чем через 10 минут после завершения задания должен загрузить на платформу фотографии черновиков с решениями заданий. Без черновиков работа не проверяется.

 

Механика

Кинематика. Механическое движение. Относительность движения.

Система отсчета. Материальная точка. Траектория. Путь и перемещение.

Скорость. Ускорение.

Равномерное и равноускоренное прямолинейное движение. Свободное падение тел. Ускорение свободного падения. Уравнение прямолинейного равноускоренного движения.

Криволинейное движение точки на примере движения по окружности с постоянной по модулю скоростью.

Центростремительное ускорение.

Основы динамики. Инерция. Первый закон Ньютона. Инерциальные системы отсчета.

Взаимодействие тел. Масса. Импульс. Сила. Второй закон Ньютона.

Принцип суперпозиции сил. Принцип относительности Галилея.

Силы в природе. Сила тяготения. Закон всемирного тяготения. Вес тела. Невесомость. Первая космическая скорость. Сила упругости. Закон

Гука. Сила трения. Коэффициент трения. Закон трения скольжения.

Третий закон Ньютона.

Момент силы. Условие равновесия тел.

Законы сохранения в механике. Закон сохранения импульса. Ракеты.

Механическая работа. Мощность. Кинетическая энергия.

Потенциальная энергия. Закон сохранения энергии в механике. Простые механизмы. Коэффициент полезного действия механизма.

Механика жидкостей и газов. Давление. Атмосферное давление.

Изменение атмосферного давления с высотой. Закон Паскаля для жидкостей и газов. Барометры и манометры. Сообщающиеся сосуды. Принцип устройства гидравлического пресса.

Архимедова сила для жидкостей и газов. Условия плавания тел на поверхности жидкости.

Движение жидкости по трубам. Зависимость давления жидкости от скорости ее течения.

Измерение расстояний, промежутков времени, силы, объема, массы, атмосферного давления.

Молекулярная физика. Термодинамика

Основы молекулярно-кинетической теории. Опытное обоснование основных положений молекулярно-кинетической теории. Броуновское движение. Диффузия. Масса и размер молекул. Измерение скорости молекул. Опыт Штерна. Количество вещества. Моль. Постоянная Авогадро.

Взаимодействие молекул. Модели газа, жидкости и твердого тела.

Основы термодинамики. Тепловое равновесие. Температура и ее измерение. Абсолютная температурная шкала. Внутренняя энергия.

Количество теплоты. Теплоемкость вещества. Работа в термодинамике.

Первый закон термодинамики. Изотермический, изохорный и изобарный процессы. Адиабатный процесс.

Необратимость тепловых процессов. Второй закон термодинамики и его статистическое истолкование. Преобразование энергии в тепловых двигателях. КПД теплового двигателя.

Идеальный газ. Связь между давлением и средней кинетической энергией молекул идеального газа. Связь температуры со средней кинетической энергией частиц газа.

Уравнение Клапейрона-Менделеева. Универсальная газовая постоянная.

Жидкости и твердые тела. Испарение и конденсация. Насыщенные и ненасыщенные пары. Влажность воздуха. Кипение жидкости.

Кристаллические и аморфные тела. Преобразование энергии при изменениях агрегатного состояния вещества.

Измерение давления газа, влажности воздуха, температуры, плотности вещества.

Основы электродинамики

Электростатика. Электризация тел. Электрический заряд.

Взаимодействие зарядов. Элементарный электрический заряд. Закон сохранения электрического заряда. Закон Кулона.

Электрическое поле. Напряженность электрического поля.

Электрическое поле точечного заряда. Потенциальность электростатического поля. Разность потенциалов. Принцип суперпозиции полей.

Проводники в электрическом поле. Электрическая емкость.

Конденсатор. Емкость плоского конденсатора.

Диэлектрики в электрическом поле. Диэлектрическая проницаемость.

Энергия электрического поля плоского конденсатора.

Постоянный электрический ток. Электрический ток. Сила тока.

Напряжение. Носители свободных электрический зарядов в металлах, жидкостях и газах. Сопротивление проводников. Закон Ома для участка цепи. Последовательное и параллельное соединение проводников.

Электродвижущая сила. Закон Ома для полной цепи. Работа и мощность тока. Закон Джоуля-Ленца.

Полупроводники. Собственная и примесная проводимость полупроводников, p-n-переход.

Магнитное поле, электромагнитная индукция. Взаимодействие магнитов. Взаимодействие проводников с током. Магнитное поле. Действие магнитного поля на электрические заряды. Индукция магнитного поля.

Сила Ампера. Сила Лоренца. Магнитный поток. Электродвигатель.

Электромагнитная индукция. Закон электромагнитной индукции

Фарадея. Правило Ленца. Вихревое электрическое поле. Самоиндукция.

Индуктивность. Энергия магнитного поля.

Измерение силы тока, напряжения, сопротивления проводника.

Колебания и волны

Механические колебания и волны. Гармонические колебания.

Амплитуда, период и частота колебаний. Свободные колебания.

Математический маятник. Период колебаний математического маятника.

Превращение энергии при гармонических колебаниях. Вынужденные колебания. Резонанс. Понятие об автоколебаниях.

Механические волны. Скорость распространения волны. Длина волны.

Поперечные и продольные волны. Уравнение гармонической волны.

Звук.

Электромагнитные колебания и волны. Колебательный контур.

Свободные электромагнитные колебания в контуре. Превращение энергии в колебательном контуре. Собственная частота колебаний в контуре.

Вынужденные электрические колебания. Переменный электрический ток.

Генератор переменного тока. Действующие значения силы тока и напряжения. Активное, емкостное и индуктивное сопротивления. Резонанс в электрической цепи.

Трансформатор. Производство, передача и потребление электрической энергии.

Идеи теории Максвелла. Электромагнитные волны. Скорость распространения электромагнитных волн. Свойства электромагнитных волн.

Принципы радиосвязи. Шкала электромагнитных волн.

Оптика

Свет — электромагнитная волна. Прямолинейное распространение, отражение и преломление света. Луч. Законы отражения и преломления света. Показатель преломления. Полное отражение. Предельный угол полного отражения. Ход лучей в призме. Построение изображений в плоском зеркале.

Собирающая и рассеивающая линзы. Формула тонкой линзы.

Построение изображений в линзах. Фотоаппарат. Глаз. Очки.

Интерференция света. Когерентность. Дифракция света. Дифракционная решетка. Поляризация света. Поперечность световых волн.

Дисперсия света.

Измерение фокусного расстояния собирающей линзы, показателя преломления вещества, длины волны света.

Основы специальной теории относительности

Инвариантность скорости света. Принцип относительности Эйнштейна.

Пространство и время в специальной теории относительности. Связь массы и энергии.

Квантовая физика

Тепловое излучение. Постоянная Планка. Фотоэффект. Опыты

Столетова. Уравнение Эйнштейна для фотоэффекта.

Гипотеза Луи де Бройля. Дифракция электронов. Корпускулярно- волновой дуализм.

Радиоактивность. Альфа-, бета-, гамма-излучения. Методы наблюдения и регистрации частиц в ядерной физике.

Опыт Резерфорда по рассеянию альфа-частиц. Планетарная модель атома. Боровская модель атома водорода. Спектры. Люминесценция.

Лазеры.

Закон радиоактивного распада. Нуклонная модель ядра. Заряд ядра.

Массовое число ядра. Энергия связи частиц в ядре. Деление ядер. Синтез ядер. Ядерные реакции. Сохранение заряда и массового числа при ядерных реакциях. Выделение энергии при делении и синтезе ядер. Использование ядерной энергии. Дозиметрия.

Элементарные частицы. Фундаментальные взаимодействия.

Методы научного познания и физическая картина мира

Эксперимент и теория в процессе познания мира. Моделирование явлений и объектов природы. Научные гипотезы. Физические законы и границы их применимости. Роль математики в физике. Принцип соответствия. Принцип причинности. Физическая картина мира.

Лаборатория физических измерений 2:

Равномерное и ускоренное движение

Введение :

Цель этого эксперимента заключается в изучении понятий средней скорости, ускорения, равномерное движение и неравномерное движение по прямой или в одном измерении. Вы также получите некоторый опыт в работа с данными.

Средняя скорость движущегося объекта за время т

(1)

В случае постоянного ускорения скорость (такая же, как скорость для линейного движения в одном направлении) изменяется на постоянную величину в равные промежутки времени.

разгон = (2)

и

(3)

Оборудование :

динамический тележка, таймер зажигания, гусеница тележки, шкив и груз.

ЭКСПЕРИМЕНТ:

A. Постоянная Скорость .

1. Отрежьте один метр длинную бегущую ленту и пропустите ее через искровой таймер.Прикрепите бегущую ленту к динамической тележке. Таймер и тележка должны быть правильно выровнены и должны быть на одном уровне. Лента бегущей строки должна быть параллельна столешнице.

2. Установить свечу таймер в режиме 10 Гц. Установите тележку рядом с таймером и слегка толкните тележку сразу после таймера. включен.Вам следует практиковать пару раз, прежде чем включить таймер зажигания. Обратите внимание, что 10 Гц означает 10 циклов в секунду и время одного цикла. составляет 0,1 сек. Поэтому время Таким образом, интервал между последовательными точками на тикерной ленте составляет 0,1 с (D t в уравнении 1 составляет 0,1 с).

3. Измерьте и запись D x , длина каждого интервала между две последовательные искровые метки. Сделать в минимум 6-7 последовательных измерений. Делать не используйте первые несколько точек.


4. Рассчитайте скорость для каждого интервала, используя уравнение (1).

5. Запишите свой результаты D t , D x и v av in таблицу и рассчитайте общую среднюю скорость путем усреднения всех v av s.

Пример

D т (с)

D x (см)

v средн. (см / с)

0.1

1,1

11

0,1

1,2

12

0,1

1,1

11

0.1

1,1

11

общая в пр. = 11.25

B. Постоянное ускорение .

1. Используйте то же самое настройку как в процедуре A. 1. Подключите тележка массой 50 грамм (только вешалка, вес добавлять не нужно) который проходит над шкивом, как показано ниже.

2.Осторожно отпустите тележка сразу после включения таймера зажигания и поймать тележку перед ним попадает в шкив.

3. Измерьте и запишите расстояние между последовательными следами искры как D x 1 , D x 2 , и т.д. Пренебрегайте первым интервалом.

Δx 2 Δx 4

Δx 1 Δx 3

4.Рассчитайте скорость для каждого интервала: v i = D x i / D t i где i = 1, 2, 3 ….. и определите другие значения для таблицы ниже


Пример , как подготовить таблицу данных:

1

2

3

4

5

6

7

D т (с)

т (т)

Δx i (см)

общее расстояние x (см)

v i (см / с)

D v (см / с)

a = (см / с 2 )

0.0

0,0

0,1

0,1

0,5

0,5

v 1 = 0.5 / 0,1 = 5,0

0,1

0,2 ​​

0,9

1,4 + 0,5 = 1,4

v 2 = 0,9 / 0,1 = 9,0

в 2 в 1 = 4.0

0,1

0,3

1,2

1,2 + 1,4 = 2,6

против 3 = 1,2 / 0,1 = 12,0

v 3 -v 2 = 3,0

30.

0,1

0,4

1,6

1,6 + 2,6 = 4,2

против 4 = 1,6 / 0,1 = 16,0

в 4 в 3 = 4,0

40.

0,1

0,5

2

2 + 4,2 = 6,2

v 5 = 2,0 / 0,1 = 20,0

в 5 в 4 = 4,0

40.

0,1

0,6

2,4

2,4 + 6,2 = 8,6

v 6 = 2,4 / 0,1 = 24,0

в 6 в 5 = 4,0

40.

Пример готовой таблицы:

т (с)

Δ x (см)

x (см)

v (см / с) = Δ x / Δ t

Δ v (см / с)

a (см / с / с) = Δ v / Δ t

0

0

0.1

0,5

0,5

5

0,2

0,9

1,4

9

4.0

40

0,3

1,2

2,6

12

3,0

30

0,4

1.6

4,2

16

4,0

40

0,5

2

6,2

20

4.0

40

0,6

2,4

8,6

24

4,0

40

5.Постройте следующие графики на миллиметровой бумаге, используя значения из ваша таблица данных выше.

а) всего расстояние (столбец 4) от общего времени (2)

b) Мгновенная скорость в зависимости от график времени (это гистограмма, как показано ниже). Обратите внимание, что для постоянного ускоренного движения мгновенное значение

скорость посреди времени interval — средняя скорость для этого временного интервала . Для получения мгновенной скорости график зависимости от времени, просто отметьте среднюю точку вверху каждой полосы и соедините их лучшей прямой линией возможно. Используйте миллиметровую бумагу (спросите инструктор копию), или компьютер для создания графика.

в) ускорение (7) от общего времени (2)

6. Определите ускорение a, найдя крутизну вашего графика скорость-время (b).

7. Сравните его со средним значением ускорения, определенным в Ваш стол для раздела Б.

8. Рассчитайте теоретическое значение (см. Свои конспекты лекций, мы прорабатывали это на примере) разгона и сравниваем с крутизной графа v-t. Какой из них меньше? Почему?

6.2: Равномерно ускоренное движение — Physics LibreTexts

Перед изучением движения в сопротивляющейся среде может быть уместно сделать краткий обзор равномерно ускоряющегося движения.То есть движение, в котором сопротивление равно нулю. Любые формулы, которые мы разрабатываем для движения в сопротивляющейся среде, должны переходить к формулам для равноускоренного движения, когда сопротивление приближается к нулю.

Можно представить себе ситуацию, в которой тело начинает со скоростью \ (v_ {0} \), а затем ускоряется со скоростью \ (a \). Можно задать три вопроса:

Как быстро он движется после времени \ (t \)?
Как далеко он продвинулся во времени \ (t \)?
Как быстро он движется после того, как преодолел расстояние \ (x \) ?

Ответы на эти вопросы хорошо известны любому изучающему физику:

\ [\ v = v_ {0} + at, \ tag {6.2 + 2акс. \ tag {6.2.3} \ label {eq: 6.2.3} \]

Поскольку ускорение равномерное, нет необходимости использовать исчисление для его получения. Первое следует непосредственно из значения ускорения. Пройденное расстояние — это площадь под графиком скорость: время. На рисунке VI.1 показан график скорость: время для постоянного ускорения, а уравнение \ (\ ref {eq: 6.2.2} \) очевидно с первого взгляда на график. Уравнение \ (\ ref {eq: 6.2.3} \) может быть получено путем исключения \ (t \) между уравнениями \ (\ ref {eq: 6.2.1} \) и \ (\ ref {eq: 6.2.2} \). (Это также можно вывести из соображений энергии, хотя это скорее ставит телегу впереди лошади.)

Тем не менее, хотя в расчетах нет необходимости, поучительно посмотреть, как их можно использовать для анализа равномерно ускоренного движения, поскольку расчет будет необходим в менее простых ситуациях. Мы будем использовать исчисление, чтобы ответить на три вопроса, поставленных ранее в этом разделе.

Для равноускоренного движения Уравнение движения равно

.

\ [\ \ ddot {x} = а.2. \ tag {6.2.6} \ label {eq: 6.2.6} \]

Это второй временной интеграл .
Чтобы получить ответ на третий вопрос, который будет называться пространственным интегралом , мы должны не забыть записать \ (\ ddot {x} \) как \ (v \ frac {dv} {dt} \ ). Таким образом, Уравнение движения (Уравнение \ (\ ref {eq: 6.2.4} \)) равно

.

\ [\ v \ frac {dv} {dx} = а. \ tag {6.2.7} \ label {eq: 6. 2_ {0} + 2акс.\ tag {6.2.8} \ label {eq: 6.2.8} \]

Это пространственный интеграл.

Примеры.

Вот несколько быстрых примеров проблем при равномерно ускоренном движении. Вероятно, будет хорошей идеей работать с алгеброй и получить алгебраических решений для каждой задачи. То есть, даже если вам говорят, что начальная скорость составляет 15 мс -1 , назовите это \ (v_ {0} \) или, если вам сказали, что высота составляет 900 футов, назовите его \ (h \ ). Вероятно, вам будет полезно рисовать графики зависимости расстояния от времени или скорости от времени в большинстве задач.{2} = 4ac \).

Пример \ (\ PageIndex {1} \)

Тело выброшено из состояния покоя. Последняя треть расстояния до падения на землю покрывается за время T. Покажите, что время, затрачиваемое на все падение на землю, составляет 5,45T.

Пример \ (\ PageIndex {2} \)

The Lady находится в 8 метрах от автобусной остановки, когда Автобус, трогаясь с остановки на автобусной остановке, трогается с места с ускорением 0,4 м с -2 . С какой наименьшей скоростью должна бежать леди, чтобы успеть на автобус?

Ответ: 2.53мс -1 .

Пример \ (\ PageIndex {3} \)

Парашютист спускается с постоянной скоростью 10 футов в секунду. Когда она находится на высоте 900 футов, ее друг, находящийся прямо под ней, бросает ей яблоко. С какой наименьшей скоростью он должен бросить яблоко, чтобы оно дошло до нее? Сколько времени нужно, чтобы добраться до нее, на каком росте она тогда и какова относительная скорость парашютиста и яблока? Предположим, что \ (g \) = 32 фута с -2 . Пренебрегать сопротивлением воздуха для яблока (но не для парашютиста!)

Ответ: 230ft -1 , 7.5s, 825 футов, 0 футов -1 .

Пример \ (\ PageIndex {4} \)

Лунный исследователь проводит следующий эксперимент на Луне, чтобы определить там гравитационное ускорение \ (g \). Он подбрасывает лунный камень вверх с начальной скоростью 15 м с -1 . Через восемь секунд он подбрасывает еще один камень вверх с начальной скоростью 10 м с -1 . Он отмечает, что камни сталкиваются через 16,32 секунды после запуска первого камня. Вычислите g, а также высоту столкновения.

Ответ: 1,64 мс -2 , 26,4 м

Пример \ (\ PageIndex {5} \)

А. и Б. обсуждают достоинства своих автомобилей. Мистер А может разогнаться от 0 до 50 миль в час за десять секунд, а мистер Б может разогнаться от 0 до 60 миль в час за 20 секунд. Мистер Б дает мистеру А начало одной секунды. Если предположить, что каждый водитель сначала равномерно разгоняется до своей максимальной скорости, а затем движется с одинаковой скоростью, сколько времени потребуется г-ну B, чтобы догнать г-на A, и как далеко к этому времени проехали машины?

Ответ: 41 с, полмили.

Я даю следующие ответы. Дайте мне знать ([email protected]), если вы думаете, что я ошибаюсь в одном из них.

Равномерно ускоренное движение и уравнения кинематики большой пятерки — стенограмма видео и урока

Равномерно ускоренное движение

Чтобы работать с кинематикой было удобнее, мы также воспользуемся этой тактикой. На этом этапе вам должно быть комфортно решать проблемы, связанные с положением, смещением, скоростью и ускорением, самостоятельно.Итак, мы собираемся сделать еще один шаг и объединить их. Но нам нужно упростить пару вещей.

Во-первых, мы рассмотрим только те объекты, которые ускоряются с постоянной скоростью, называемой «равномерно ускоренным движением». Это редко достигается в реальном мире из-за дополнительных внешних сил, создающих изменчивость в том, насколько быстро или медленно объект ускоряется на протяжении всего своего движения. Чтобы упростить задачу, мы пока не будем возиться ни с одним из них.

Во-вторых, мы будем рассматривать только объекты, движущиеся по прямой линии.Это устраняет любые неприятные проблемы с направленным компонентом, необходимым для векторных величин и вычислений. Поскольку мы застряли на одной прямой, нам нужно беспокоиться только о направлениях вперед и назад, которые мы назовем положительными и отрицательными. Для этих задач достаточно знака. Никаких дополнительных дескрипторов, таких как север, верх или лево, не требуется.

Эти ограничения могут показаться нереальными в реальном мире, но равномерно ускоренное движение по прямой — отличный способ узнать, как концепции кинематики объединяются в пять основных уравнений.

Уравнения большой пятерки

Это уравнения большой пятерки:

В качестве напоминания я сначала определю каждую из переменных.

Символ дельта (Δ) означает «изменение».

x = конечное положение

x 0 = начальное положение

v = конечная скорость

v 0 = начальная скорость

v с полосой над ним = средняя скорость

a = ускорение

t = время

Вопросы о равномерно ускоренном движении предоставят вам некоторые из этих частей информации и попросят вас решить для неизвестной величины.Ключ состоит в том, чтобы вытащить значения и определить, какие из них у вас есть, какие нужно определить, а какие вообще не включены в вопрос. Затем просто включите их в правильное уравнение. Это может показаться простым «заполните пробелы и сделайте математику», но это может быть немного сложнее. Извините, но вы должны запомнить эти пять уравнений. Ни один из вопросов с несколькими вариантами ответов, с которыми вы столкнетесь, не даст вам их.

Здесь нужно отметить еще один момент.В некоторых вопросах может показаться, что вам не хватает ключевой информации, особенно начальной позиции. Если вы ищете изменение положения в течение определенного периода времени, и вопрос не дает вам начального положения, вы можете предположить, что оно составляет 0 метров. Не забывайте всегда перепроверять свои переменные и уравнения и будьте очень осторожны, предполагая что-либо.

Использование уравнений

Давайте рассмотрим типичную проблему, чтобы вы получили представление о том, как ее решать.

Гоночный автомобиль, стоящий на стартовой линии прямой трассы, ускоряется равномерно на 3 секунды.6 секунд со скоростью 4,5 м / с2. Если начальная скорость равна 0 м / с, как далеко проехала машина за этот промежуток времени?

Во-первых, давайте запишем переменные, которые мы заданы в уравнении.

t = 3,6 с
a = 4,5 м / с2
v 0 = 0 м / с
x 0 = 0 м. Это не указано, но вы можете предположить, что это 0 м, чтобы рассчитать изменение положения.
x = то, что проблема просит вас решить.

Теперь посмотрим на «большую пятерку» уравнений. Только один будет иметь ровно эти пять переменных. В этом случае нам понадобится уравнение 3: x = x 0 + v 0 * t + ½ на 2. Теперь начните заполнять пробелы и вычислить ответ.

Итак, в этой задаче гонщик преодолевает около 29 метров за 3,6 секунды. Вы должны указать единицы измерения, то есть метры.

Если в любой момент во время вопросов, подобных этому, вы не уверены, что у вас правильное уравнение, попробуйте подставить числа в несколько уравнений.Вы быстро обнаружите, что у вас есть неиспользованные значения или уравнению требуется значение, которого у вас нет.

Итоги урока

Давайте кратко рассмотрим.

Чтобы получить более удобное решение задач с различными комбинациями переменных положения, смещения, скорости и ускорения, мы упрощаем несколько аспектов этих задач. Во-первых, мы предполагаем, что все ускорения однородны, то есть они происходят с одинаковой скоростью от начала до конца. Во-вторых, мы смотрим только на движение по прямой линии, поэтому направление, связанное с векторными величинами, может быть только положительным или отрицательным.

Есть пять уравнений, которые вам нужно запомнить для решения задач о равномерно ускоренном движении. Лучший способ начать — определить все переменные, включая ту, которую нужно вычислить, и найти уравнение, в котором есть все из них. Затем просто подключите их и произведите вычисления. Помните, что для правильного ответа на вопрос вам необходимо указать правильные единицы измерения.

Результаты обучения

По завершении этого урока вы должны уметь:

  • Вспомнить уравнения Большой пятерки для равноускоренного движения
  • Определите переменные в уравнении равноускоренного движения
  • Решите задачу о равномерно ускоренном движении

3.4 Движение с постоянным ускорением — University Physics Volume 1

Learning Objectives

К концу этого раздела вы сможете:

  • Определите, какие уравнения движения следует использовать для решения неизвестных.
  • Используйте соответствующие уравнения движения для решения задачи о преследовании двух тел.

Вы можете догадаться, что чем больше ускорение, скажем, у автомобиля, удаляющегося от знака «Стоп», тем больше смещение автомобиля за данный момент времени.Но мы не разработали конкретное уравнение, которое связывает ускорение и смещение. В этом разделе мы рассмотрим некоторые удобные уравнения кинематических отношений, начиная с определений смещения, скорости и ускорения. Сначала мы исследуем движение одного объекта, называемого движением одного тела. Затем мы исследуем движение двух объектов, называемое задачами преследования двух тел.

Обозначение

Во-первых, сделаем несколько упрощений в обозначениях. Принятие начального времени равным нулю, как если бы время измерялось секундомером, является большим упрощением.Поскольку прошедшее время равно Δt = tf − t0Δt = tf − t0, принятие t0 = 0t0 = 0 означает, что Δt = tfΔt = tf, последнее время на секундомере. Когда начальное время принимается равным нулю, мы используем индекс 0 для обозначения начальных значений положения и скорости. То есть x0x0 — это начальная позиция , а v0v0 — начальная скорость . Мы не ставим индексы на окончательные значения. То есть t — это последнее время , x — это конечное положение , а v — это конечная скорость . Это дает более простое выражение для затраченного времени: Δt = tΔt = t.Это также упрощает выражение для смещения x , которое теперь равно Δx = x − x0Δx = x − x0. Кроме того, это упрощает выражение для изменения скорости, которое теперь равно Δv = v − v0Δv = v − v0. Подводя итог, используя упрощенные обозначения, с начальным временем, принятым равным нулю,

Δt = tΔx = x − x0Δv = v − v0, Δt = tΔx = x − x0Δv = v − v0,

, где нижний индекс 0 обозначает начальное значение, а отсутствие нижнего индекса означает конечное значение в любом рассматриваемом движении.

Теперь мы делаем важное предположение, что ускорение постоянно .Это предположение позволяет нам избегать использования расчетов для определения мгновенного ускорения. Поскольку ускорение постоянно, среднее и мгновенное ускорения равны, то есть

a– = a = постоянная. a– = a = постоянная.

Таким образом, мы можем использовать символ a для ускорения в любое время. Предположение, что ускорение является постоянным, не серьезно ограничивает ситуации, которые мы можем изучить, и не ухудшает точность нашего лечения. Во-первых, ускорение постоянно равно в большом количестве ситуаций.Кроме того, во многих других ситуациях мы можем точно описать движение, приняв постоянное ускорение, равное среднему ускорению для этого движения. Наконец, для движения, во время которого ускорение резко меняется, например, когда автомобиль разгоняется до максимальной скорости, а затем тормозит до остановки, движение можно рассматривать в отдельных частях, каждая из которых имеет собственное постоянное ускорение.

Смещение и положение от скорости

Чтобы получить наши первые два уравнения, мы начнем с определения средней скорости:

Подставляя упрощенные обозначения для ΔxΔx и ΔtΔt, получаем

v– = x − x0t.v– = x − x0t.

Решение относительно x дает нам

x = x0 + v – t, x = x0 + v – t,

3,10

при средней скорости

v– = v0 + v2.v– = v0 + v2.

3,11

Уравнение v– = v0 + v2v– = v0 + v2 отражает тот факт, что при постоянном ускорении v – v– представляет собой простое среднее значение начальной и конечной скоростей. Рисунок 3.18 графически иллюстрирует эту концепцию. В части (а) рисунка ускорение является постоянным, а скорость увеличивается с постоянной скоростью. Средняя скорость в течение 1-часового интервала от 40 км / ч до 80 км / ч составляет 60 км / ч:

v– = v0 + v2 = 40 км / ч + 80 км / ч3 = 60 км / ч.v– = v0 + v2 = 40 км / ч + 80 км / ч3 = 60 км / ч.

В части (b) ускорение не является постоянным. В течение 1-часового интервала скорость ближе к 80 км / ч, чем к 40 км / ч. Таким образом, средняя скорость больше, чем в части (а).

Рисунок 3.18 (a) График зависимости скорости от времени с постоянным ускорением, показывающий начальную и конечную скорости v0andvv0andv. Средняя скорость равна 12 (v0 + v) = 60 км / ч 22 (v0 + v) = 60 км / ч. (б) График зависимости скорости от времени с изменением ускорения со временем. Средняя скорость не равна 12 (v0 + v) 12 (v0 + v), но превышает 60 км / ч.

Решение окончательной скорости по ускорению и времени

Мы можем вывести еще одно полезное уравнение, манипулируя определением ускорения:

Подстановка упрощенных обозначений для ΔvΔv и ΔtΔt дает

а = v − v0t (константа). a = v − v0t (константа).

Решение для v дает

v = v0 + at (constanta). v = v0 + at (constanta).

3,12

Пример 3,7

Расчет конечной скорости
Самолет приземляется с начальной скоростью 70.0 м / с, а затем ускоряется против движения со скоростью 1,50 м / с 2 за 40,0 с. Какова его конечная скорость?
Стратегия
Сначала мы идентифицируем известные: v0 = 70 м / с, a = -1,50 м / с2, t = 40sv0 = 70 м / с, a = -1,50 м / с2, t = 40 с.

Во-вторых, мы идентифицируем неизвестное; в данном случае это конечная скорость vfvf.

Наконец, мы определяем, какое уравнение использовать. Для этого мы выясняем, какое кинематическое уравнение дает неизвестное в терминах известных. Мы рассчитываем окончательную скорость, используя уравнение 3.12, v = v0 + atv = v0 + at.

Решение
Подставьте известные значения и решите: v = v0 + at = 70,0 м / с + (- 1,50 м / с2) (40,0 с) = 10,0 м / сv = v0 + at = 70,0 м / с + (- 1,50 м / с2) (40,0 с) = 10,0 м / с.

Рисунок 3.19 представляет собой эскиз, показывающий векторы ускорения и скорости.

Рис. 3.19. Самолет приземляется с начальной скоростью 70,0 м / с и замедляется до конечной скорости 10,0 м / с, прежде чем направиться к терминалу. Обратите внимание, что ускорение отрицательное, потому что его направление противоположно его скорости, которая положительна.

Значение
Конечная скорость намного меньше начальной скорости, требуемой при замедлении, но все же положительная (см. Рисунок). В реактивных двигателях обратная тяга может поддерживаться достаточно долго, чтобы самолет остановился и начал движение назад, на что указывает отрицательная конечная скорость, но в данном случае это не так.

Помимо полезности при решении задач, уравнение v = v0 + atv = v0 + at дает нам представление о взаимосвязях между скоростью, ускорением и временем.Мы видим, например, что

  • Конечная скорость зависит от того, насколько велико ускорение и как долго оно длится
  • Если ускорение равно нулю, то конечная скорость равна начальной скорости ( v = v 0 ), как и ожидалось (другими словами, скорость постоянна)
  • Если a отрицательное, то конечная скорость меньше начальной скорости

Все эти наблюдения соответствуют нашей интуиции. Обратите внимание, что всегда полезно исследовать основные уравнения в свете нашей интуиции и опыта, чтобы убедиться, что они действительно точно описывают природу.

Решение для конечного положения с постоянным ускорением

Мы можем объединить предыдущие уравнения, чтобы найти третье уравнение, которое позволяет нам вычислить окончательное положение объекта, испытывающего постоянное ускорение. Начнем с

Добавление v0v0 к каждой стороне этого уравнения и деление на 2 дает

v0 + v2 = v0 + 12at. v0 + v2 = v0 + 12at.

Так как v0 + v2 = v – v0 + v2 = v– для постоянного ускорения, имеем

v– = v0 + 12at.v– = v0 + 12at.

Теперь мы подставляем это выражение для v – v– в уравнение для смещения, x = x0 + v – tx = x0 + v – t, что дает

х = х0 + v0t + 12at2 (константа).х = х0 + v0t + 12at2 (константа).

3,13

Пример 3.8

Расчет смещения ускоряющегося объекта
Драгстеры могут развивать среднее ускорение 26,0 м / с 2 . Предположим, драгстер ускоряется из состояния покоя с этой скоростью в течение 5,56 с. Рис. 3.20. Как далеко он пролетит за это время?

Рис. 3.20. Пилот Top Fuel в армии США Тони «Сержант» Шумахер начинает гонку с контролируемого выгорания. (Источник: подполковник Уильям Термонд. Фото любезно предоставлено США.Армия.)

Стратегия
Сначала нарисуем набросок Рис. 3.21. Нас просят найти смещение, которое составляет x , если мы возьмем x0x0 равным нулю. (Думайте о x0x0 как о стартовой линии гонки. Она может быть где угодно, но мы называем ее нулевой и измеряем все остальные позиции относительно нее.) Мы можем использовать уравнение x = x0 + v0t + 12at2x = x0 + v0t + 12at2 когда мы идентифицируем v0v0, aa и t из постановки задачи.

Рис. 3.21. Эскиз разгоняющегося драгстера.

Решение
Во-первых, нам нужно определить известные.Запуск из состояния покоя означает, что v0 = 0v0 = 0, a задается как 26,0 м / с 2 и t задается как 5,56 с.

Во-вторых, мы подставляем известные значения в уравнение, чтобы найти неизвестное:

x = x0 + v0t + 12at2.x = x0 + v0t + 12at2.

Поскольку начальное положение и скорость равны нулю, это уравнение упрощается до

Подстановка идентифицированных значений на и t дает

x = 12 (26,0 м / с2) (5,56 с) 2 = 402 м. x = 12 (26,0 м / с2) (5,56 с) 2 = 402 м.
Значение
Если мы переведем 402 м в мили, мы обнаружим, что пройденное расстояние очень близко к четверти мили, стандартному расстоянию для дрэг-рейсинга. Итак, наш ответ разумный. Это впечатляющий водоизмещение всего за 5,56 с, но первоклассные драгстеры могут преодолеть четверть мили даже за меньшее время. Если бы драгстеру была присвоена начальная скорость, это добавило бы еще один член в уравнение расстояния. Если в уравнении использовать те же ускорение и время, пройденное расстояние будет намного больше.

Что еще мы можем узнать, исследуя уравнение x = x0 + v0t + 12at2? X = x0 + v0t + 12at2? Мы видим следующие отношения:

  • Смещение зависит от квадрата прошедшего времени, когда ускорение не равно нулю. В примере 3.8 драгстер преодолевает только четверть общего расстояния за первую половину прошедшего времени.
  • Если ускорение равно нулю, то начальная скорость равна средней скорости (v0 = v -) (v0 = v–), и x = x0 + v0t + 12at2becomesx = x0 + v0t.x = x0 + v0t + 12at2becomesx = x0 + v0t.

Расчет конечной скорости по расстоянию и ускорению

Четвертое полезное уравнение может быть получено путем другой алгебраической обработки предыдущих уравнений. Если мы решим v = v0 + atv = v0 + at для t , мы получим

Подставляя это и v– = v0 + v2v– = v0 + v2 в x = x0 + v – tx = x0 + v – t, получаем

v2 = v02 + 2a (x − x0) (constanta). v2 = v02 + 2a (x − x0) (constanta).

3,14

Пример 3.9

Расчет конечной скорости
Рассчитайте конечную скорость драгстера в Примере 3.8 без использования информации о времени.
Стратегия
Уравнение v2 = v02 + 2a (x − x0) v2 = v02 + 2a (x − x0) идеально подходит для этой задачи, поскольку оно связывает скорости, ускорение и смещение и не требует информации о времени.
Решение
Сначала мы идентифицируем известные значения. Мы знаем, что v 0 = 0, поскольку драгстер запускается из состояния покоя. Мы также знаем, что x x 0 = 402 м (это был ответ в примере 3.8). Среднее ускорение было равно , а = 26.0 м / с 2 .

Во-вторых, мы подставляем известные значения в уравнение v2 = v02 + 2a (x − x0) v2 = v02 + 2a (x − x0) и решаем относительно v :

v2 = 0 + 2 (26,0 м / с2) (402 м). v2 = 0 + 2 (26,0 м / с2) (402 м).

Таким образом,

v2 = 2,09 × 104 м2 / с2 v = 2,09 × 104 м2 / с2 = 145 м / с. v2 = 2,09 × 104 м2 / с2v = 2,09 × 104 м2 / с2 = 145 м / с.
Значение
Скорость 145 м / с составляет около 522 км / ч или около 324 миль / ч, но даже эта головокружительная скорость не достигает рекорда для четверти мили. Также обратите внимание, что квадратный корень имеет два значения; мы взяли положительное значение, чтобы указать скорость в том же направлении, что и ускорение.

Изучение уравнения v2 = v02 + 2a (x − x0) v2 = v02 + 2a (x − x0) может дать дополнительное понимание общих соотношений между физическими величинами:

  • Конечная скорость зависит от величины ускорения и расстояния, на котором оно действует.
  • При фиксированном ускорении автомобиль, который едет вдвое быстрее, не просто останавливается на удвоенном расстоянии. Чтобы остановиться, нужно гораздо дальше. (Вот почему у нас есть зоны с пониженной скоростью возле школ.)

Объединение уравнений

В следующих примерах мы продолжаем исследовать одномерное движение, но в ситуациях, требующих немного большего количества алгебраических манипуляций.Примеры также дают представление о методах решения проблем. Следующее примечание предназначено для облегчения поиска необходимых уравнений. Имейте в виду, что эти уравнения не являются независимыми. Во многих ситуациях у нас есть два неизвестных, и нам нужно два уравнения из набора для решения неизвестных. Для решения данной ситуации нам нужно столько уравнений, сколько неизвестных.

Сводка кинематических уравнений (константа

a ) х = х0 + v0t + 12at2x = x0 + v0t + 12at2 v2 = v02 + 2a (x − x0) v2 = v02 + 2a (x − x0)

Прежде чем мы перейдем к примерам, давайте более внимательно рассмотрим некоторые уравнения, чтобы увидеть поведение ускорения при экстремальных значениях.Переставляя уравнение 3.12, получаем

Из этого мы видим, что в течение конечного времени, если разница между начальной и конечной скоростями мала, ускорение невелико, приближаясь к нулю в пределе, когда начальная и конечная скорости равны. Напротив, в пределе t → 0t → 0 при конечной разности начальной и конечной скоростей ускорение становится бесконечным.

Аналогичным образом, переставляя уравнение 3.14, мы можем выразить ускорение в терминах скоростей и смещения:

а = v2-v022 (х-х0).а = v2-v022 (х-х0).

Таким образом, при конечной разнице между начальной и конечной скоростями ускорение становится бесконечным, в пределе смещение приближается к нулю. Ускорение приближается к нулю в пределе, разница в начальной и конечной скоростях приближается к нулю для конечного смещения.

Пример 3.10

Как далеко уезжает машина?
На сухом бетоне автомобиль может ускоряться противоположно движению со скоростью 7,00 м / с 2 , тогда как на мокром бетоне он может ускоряться противоположно движению со скоростью всего 5 м / с.00 м / с 2 . Найдите расстояния, необходимые для остановки автомобиля, движущегося со скоростью 30,0 м / с (около 110 км / ч) по (а) сухому бетону и (б) мокрому бетону. (c) Повторите оба вычисления и найдите смещение от точки, где водитель видит, что светофор становится красным, принимая во внимание время его реакции 0,500 с, чтобы нажать ногой на тормоз.
Стратегия
Для начала нам нужно нарисовать набросок Рис. 3.22. Чтобы определить, какие уравнения лучше всего использовать, нам нужно перечислить все известные значения и точно определить, что нам нужно решить.

Рис. 3.22. Пример эскиза для визуализации ускорения, противоположного движению и тормозному пути автомобиля.

Решение
  1. Во-первых, нам нужно определить известные и то, что мы хотим решить. Мы знаем, что v 0 = 30,0 м / с, v = 0 и a = −7,00 м / с 2 ( a отрицательно, потому что оно находится в направлении, противоположном скорости) . Возьмем x 0 равным нулю. Ищем смещение ΔxΔx, или x x 0 .
    Во-вторых, мы определяем уравнение, которое поможет нам решить проблему. Лучшее уравнение для использования — v2 = v02 + 2a (x − x0). v2 = v02 + 2a (x − x0). Это уравнение лучше всего, потому что оно включает только одно неизвестное, x . Нам известны значения всех других переменных в этом уравнении. (Другие уравнения позволили бы нам решить для x , но они требуют, чтобы мы знали время остановки, t , которое мы не знаем. Мы могли бы их использовать, но это потребовало бы дополнительных вычислений.)
    В-третьих, мы изменим уравнение, чтобы найти x : x − x0 = v2 − v022ax − x0 = v2 − v022a и подставляем известные значения: х − 0 = 02− (30.0 м / с) 22 (-7,00 м / с2). X-0 = 02- (30,0 м / с) 22 (-7,00 м / с2). Таким образом, x = 64,3 м на сухом бетоне. x = 64,3 м на сухом бетоне.
  2. Эта часть может быть решена точно так же, как (а). Единственное отличие состоит в том, что ускорение составляет −5,00 м / с 2 . Результат xwet = 90,0 м по мокрому бетону. xwet = 90,0 м по мокрому бетону.
  3. Когда водитель реагирует, тормозной путь такой же, как в (a) и (b) для сухого и влажного бетона. Итак, чтобы ответить на этот вопрос, нам нужно вычислить, как далеко проехал автомобиль за время реакции, а затем добавить это время ко времени остановки.Разумно предположить, что скорость остается постоянной в течение времени реакции водителя.
    Для этого мы снова определяем известные и то, что мы хотим решить. Мы знаем, что v– = 30,0 м / sv– = 30,0 м / с, treaction = 0,500streaction = 0,500s и areaction = 0areaction = 0. Примем x0-реакцию x0-реакцию равной нулю. Ищем xreactionxreaction.
    Во-вторых, как и раньше, мы определяем лучшее уравнение для использования. В этом случае x = x0 + v – tx = x0 + v – t работает хорошо, потому что единственное неизвестное значение — x , что мы и хотим найти.
    В-третьих, мы подставляем известные для решения уравнения: x = 0 + (30,0 м / с) (0,500 с) = 15,0 м. x = 0 + (30,0 м / с) (0,500 с) = 15,0 м. Это означает, что автомобиль проезжает 15,0 м, в то время как водитель реагирует, в результате чего общее смещение в двух случаях с сухим и мокрым бетоном на 15,0 м больше, чем если бы он реагировал мгновенно.
    Наконец, мы добавляем смещение во время реакции к смещению при торможении (рис. 3.23), xbraking + xreaction = xtotal, xbraking + xreaction = xtotal, и найдите (a) равным 64,3 м + 15,0 м = 79.3 м в сухом состоянии и (b) должно составлять 90,0 м + 15,0 м = 105 м во влажном состоянии.

Рис. 3.23. Расстояние, необходимое для остановки автомобиля, сильно различается в зависимости от дорожных условий и времени реакции водителя. Здесь показаны значения тормозного пути для сухого и мокрого покрытия, рассчитанные в этом примере для автомобиля, движущегося со скоростью 30,0 м / с. Также показаны общие расстояния, пройденные от точки, когда водитель впервые видит, что свет загорается красным, при условии, что время реакции составляет 0,500 с.

Значение
Смещения, обнаруженные в этом примере, кажутся разумными для остановки быстро движущегося автомобиля.Остановка автомобиля на мокром асфальте должна длиться дольше, чем на сухом. Интересно, что время реакции значительно увеличивает смещения, но более важен общий подход к решению проблем. Мы идентифицируем известные и определяемые величины, а затем находим соответствующее уравнение. Если существует более одного неизвестного, нам нужно столько независимых уравнений, сколько неизвестных необходимо решить. Часто есть несколько способов решить проблему. Фактически, различные части этого примера могут быть решены другими методами, но представленные здесь решения являются самыми короткими.

Пример 3.11

Время расчета
Предположим, автомобиль въезжает в движение по автостраде на съезде длиной 200 м. Если его начальная скорость равна 10,0 м / с, а он ускоряется со скоростью 2,00 м / с 2 , сколько времени потребуется автомобилю, чтобы преодолеть 200 м по рампе? (Такая информация может быть полезна транспортному инженеру.)
Стратегия
Сначала мы рисуем набросок Рис. 3.24. Нам предлагается решить за время т . Как и раньше, мы идентифицируем известные величины, чтобы выбрать удобную физическую связь (то есть уравнение с одной неизвестной, t .)

Рис. 3.24. Эскиз автомобиля, ускоряющегося на съезде с автострады.

Решение
Опять же, мы идентифицируем то, что нам известно, и то, что мы хотим решить. Мы знаем, что x0 = 0, x0 = 0,
v0 = 10 м / с, a = 2,00 м / с2v0 = 10 м / с, a = 2,00 м / с2 и x = 200 м.

Нам нужно решить для т . Уравнение x = x0 + v0t + 12at2x = x0 + v0t + 12at2 работает лучше всего, потому что единственной неизвестной в уравнении является переменная t , которую нам нужно решить. Из этого понимания мы видим, что когда мы вводим известные значения в уравнение, мы получаем квадратное уравнение.

Нам нужно изменить уравнение, чтобы найти t , затем подставив известные значения в уравнение:

200 м = 0 м + (10,0 м / с) t + 12 (2,00 м / с2) t2. 200 м = 0 м + (10,0 м / с) t + 12 (2,00 м / с2) t2.

Затем мы упрощаем уравнение. Единицы измерения отменяются, потому что они есть в каждом члене. Мы можем получить единицы секунд для отмены, взяв t = t с, где t — величина времени, а s — единица измерения. Остается

Затем мы используем формулу корней квадратного уравнения, чтобы найти t ,

t2 + 10t − 200 = 0t = −b ± b2−4ac2a, t2 + 10t − 200 = 0t = −b ± b2−4ac2a,

, что дает два решения: t = 10.0 и t = −20,0. Отрицательное значение времени неразумно, так как это будет означать, что событие произошло за 20 секунд до начала движения. Мы можем отказаться от этого решения. Таким образом,

Значение
Всякий раз, когда уравнение содержит неизвестный квадрат, есть два решения. В некоторых проблемах имеют смысл оба решения; в других случаях разумно только одно решение. Ответ 10,0 с кажется разумным для типичной автострады на съезде.

Проверьте свое понимание 3.5

Ракета ускоряется со скоростью 20 м / с 2 во время пуска.Сколько времени нужно, чтобы ракета достигла скорости 400 м / с?

Пример 3.12

Ускорение космического корабля
Космический корабль покинул орбиту Земли и направляется к Луне. Разгоняется со скоростью 20 м / с 2 за 2 мин и преодолевает расстояние до 1000 км. Каковы начальная и конечная скорости космического корабля?
Стратегия
Нас просят найти начальную и конечную скорости космического корабля. Глядя на кинематические уравнения, мы видим, что одно уравнение не дает ответа.Мы должны использовать одно кинематическое уравнение для решения одной из скоростей и подставить его в другое кинематическое уравнение, чтобы получить вторую скорость. Таким образом, мы решаем два кинематических уравнения одновременно.
Решение
Сначала мы решаем для v0v0, используя x = x0 + v0t + 12at2: x = x0 + v0t + 12at2: x − x0 = v0t + 12at2x − x0 = v0t + 12at21.0 × 106m = v0 (120.0s) +12 (20,0 м / с2) (120,0 с) 21,0 × 106 м = v0 (120,0 с) +12 (20,0 м / с2) (120,0 с) 2v0 = 7133,3 м / с. V0 = 7133,3 м / с.

Затем мы подставляем v0v0 в v = v0 + atv = v0 + at, чтобы найти окончательную скорость:

v = v0 + at = 7133.3 м / с + (20,0 м / с2) (120,0 с) = 9533,3 м / с. V = v0 + at = 7133,3 м / с + (20,0 м / с2) (120,0 с) = 9533,3 м / с.
Значение
Есть шесть переменных смещения, времени, скорости и ускорения, которые описывают движение в одном измерении. Начальные условия данной задачи могут быть множеством комбинаций этих переменных. Из-за такого разнообразия решения могут быть не такими простыми, как простая подстановка в одно из уравнений. Этот пример показывает, что решения кинематики могут потребовать решения двух одновременных кинематических уравнений.

Освоив основы кинематики, мы можем перейти ко многим другим интересным примерам и приложениям. В процессе разработки кинематики мы также увидели общий подход к решению проблем, который дает как правильные ответы, так и понимание физических взаимоотношений. Следующий уровень сложности в наших задачах кинематики включает движение двух взаимосвязанных тел, называемых задачами преследования двух тел .

Задачи преследования двух тел

До этого момента мы рассматривали примеры движения с участием одного тела.Даже для задачи с двумя автомобилями и тормозным путем на мокрой и сухой дороге мы разделили эту задачу на две отдельные задачи, чтобы найти ответы. В задаче преследования двух тел движения объектов связаны, то есть искомое неизвестное зависит от движения обоих объектов. Чтобы решить эти проблемы, мы пишем уравнения движения для каждого объекта, а затем решаем их одновременно, чтобы найти неизвестное. Это показано на Рисунке 3.25.

Рис. 3.25. Сценарий преследования с двумя телами, в котором автомобиль 2 имеет постоянную скорость, а автомобиль 1 идет сзади с постоянным ускорением.Автомобиль 1 догонит автомобиль 2 позже.

Время и расстояние, необходимое для того, чтобы автомобиль 1 догнал автомобиль 2, зависит от начального расстояния, на которое автомобиль 1 находится от автомобиля 2, а также от скорости обоих автомобилей и ускорения автомобиля 1. Кинематические уравнения, описывающие движение обоих автомобилей, должны быть решил найти эти неизвестные.

Рассмотрим следующий пример.

Пример 3.13

Гепард ловит газель
Гепард прячется за кустом. Гепард замечает пробегающую мимо газель со скоростью 10 м / с.В тот момент, когда газель проходит мимо гепарда, гепард из состояния покоя ускоряется со скоростью 4 м / с 2 , чтобы поймать газель. а) Сколько времени требуется гепарду, чтобы поймать газель? б) Что такое смещение газели и гепарда?
Стратегия
Мы используем систему уравнений для постоянного ускорения, чтобы решить эту проблему. Поскольку есть два движущихся объекта, у нас есть отдельные уравнения движения, описывающие каждое животное. Но то, что связывает уравнения, — это общий параметр, который имеет одинаковое значение для каждого животного.Если мы внимательно посмотрим на проблему, становится ясно, что общим параметром для каждого животного является их положение x в более позднее время t . Поскольку оба они начинаются с x0 = 0x0 = 0, их смещения будут такими же в более позднее время t , когда гепард догонит газель. Если мы выберем уравнение движения, которое решает проблему смещения для каждого животного, мы сможем приравнять уравнения друг к другу и решить неизвестное, то есть время.
Решение
  1. Уравнение для газели: Газель имеет постоянную скорость, которая является ее средней скоростью, поскольку она не ускоряется.Поэтому мы используем уравнение 3.10 с x0 = 0x0 = 0: x = x0 + v – t = v – t. x = x0 + v – t = v – t. Уравнение для гепарда: гепард ускоряется из состояния покоя, поэтому мы используем уравнение 3.13 с x0 = 0x0 = 0 и v0 = 0v0 = 0: x = x0 + v0t + 12at2 = 12at2.x = x0 + v0t + 12at2 = 12at2. Теперь у нас есть уравнение движения для каждого животного с общим параметром, который можно исключить, чтобы найти решение. В этом случае мы решаем для t : x = v – t = 12at2t = 2v – a.x = v – t = 12at2t = 2v – a. Газель имеет постоянную скорость 10 м / с, что составляет ее среднюю скорость.Ускорение гепарда составляет 4 м / с 2 . Оценив t , время, за которое гепард достигнет газели, имеем t = 2v – a = 2 (10 м / с) 4m / s2 = 5s. t = 2v – a = 2 (10 м / с) 4m / s2 = 5s.
  2. Чтобы получить смещение, мы используем уравнение движения гепарда или газели, поскольку оба они должны дать одинаковый ответ.
    Смещение гепарда: x = 12at2 = 12 (4 м / с2) (5) 2 = 50 м. x = 12at2 = 12 (4 м / с2) (5) 2 = 50 м. Водоизмещение газели: x = v – t = 10 м / с (5) = 50 м. x = v – t = 10 м / с (5) = 50 м.Мы видим, что оба смещения равны, как и ожидалось.
Значение
Важно анализировать движение каждого объекта и использовать соответствующие кинематические уравнения для описания отдельного движения. Также важно иметь хорошую визуальную перспективу задачи преследования двух тел, чтобы увидеть общий параметр, который связывает движение обоих объектов.

Проверьте свое понимание 3.6

Велосипед имеет постоянную скорость 10 м / с. Человек начинает с отдыха и начинает бежать, чтобы догнать велосипед через 30 секунд, когда велосипед находится в том же положении, что и человек.Какое ускорение у человека?

Galileo и Free Fall

Если гвоздь и зубочистка одновременно упали с одного и того же высоты, они не достигают земли в одно и то же время. (Пытаться с этими или подобными объектами.)

Две новые науки имеют дело непосредственно с движением свободно падающих тела. Изучая следующие абзацы из него, мы должны быть начеку. к общему плану Галилея. Во-первых, он обсуждает математику возможный, простой тип движения (который мы теперь называем равномерным ускорением или постоянное ускорение).Затем он предлагает, чтобы на самом деле тяжелые тела падать именно так. Затем на основе этого предложения он выводит предсказание о скатывании мячей по склону. Наконец, он показывает что эксперименты подтверждают эти прогнозы. По аристотелевской космологии имеется в виду весь взаимосвязанный набор представлений о структуре физическая вселенная и поведение всех объектов в ней.

На самом деле было сделано больше, чем просто «поверхностные наблюдения». задолго до того, как Галилей приступил к работе.Например, Николас Орем и другие. в Парижском университете к 1330 г. открыли такое же расстояние соотношение времени для падающих тел, которое Галилей должен был объявить в Две новые науки. Это поможет вам иметь четкий план по мере продвижения по оставшейся части этой главы. Когда вы изучаете каждый в следующем разделе спросите себя, является ли Галилео


(1) «равномерное» ускорение означает равные приращения скорости, Dv, через равные промежутки времени, Dt; и
(2) на самом деле все так и происходит.

Давайте сначала посмотрим более внимательно на определение, предложенное Галилеем.

Это единственно возможный способ определения равномерного ускорения? Нисколько! Галилей говорит, что когда-то считал более полезным определение будет использовать термин равномерное ускорение для движения в скорость которой увеличивалась пропорционально пройденному расстоянию Dd, а не соответствию времени. Обратите внимание, что оба определения отвечали требованиям простоты Галилея.(Фактически, оба определения обсуждались с начала четырнадцатого века.)
Более того, оба определения, похоже, соответствуют нашему здравому смыслу представления о разгон примерно одинаково хорошо. Когда мы говорим, что тело «ускорение», мы, кажется, подразумеваем «чем дальше он идет, тем быстрее идет «, а также» чем дольше он идет, тем быстрее идет «. Как должны ли мы выбирать между этими двумя способами выражения? Какое определение будет полезнее в описании природы? Вот где экспериментирование становится важным.Галилей решил определить равномерное ускорение как движение, при котором изменение скорости v пропорционально истекшему времени Dt, и затем продемонстрируйте, что это соответствует поведению реальных движущихся тел, в лабораторных условиях, а также в обычных, «неорганизованных», опыт. Как вы увидите позже, он сделал правильный выбор. Но он был не в состоянии доказать свою правоту прямыми или очевидными способами, поскольку вы должны также см.
Опишите равномерную скорость без упоминания шайб сухого льда и стробоскопическая фотография или художественный конкретный объект или техника измерение.
Выразите определение равномерно ускоренного движения, данное Галилеем. словами и в виде уравнения.
Какие два условия хотел Галилей своим определением униформы? ускорение встретить?

Галилей не может напрямую проверить свою гипотезу

После того, как Галилей определил равномерное ускорение, чтобы оно соответствовало в как он считал себя свободно падающими объектами, его следующей задачей было придумать способ показать, что определение равномерного ускорения был полезен для описания наблюдаемых движений.

Предположим, мы уронили тяжелый предмет с разной высоты. скажем, из окон на разных этажах здания. Мы хотим проверить увеличивается ли конечная скорость пропорционально времени, необходимому для падение — то есть, является ли Dv «пропорциональным на «Dt» или что то же самое, является ли Dv / Dt постоянный. В каждом испытании мы должны соблюдать время падения и скорость непосредственно перед тем, как объект ударится о землю.

Но вот загвоздка.Практически даже сегодня было бы очень трудно произвести прямое измерение скорости, достигаемой объект непосредственно перед тем, как удариться о землю. Кроме того, все время интервалы падения (менее 3 секунд даже с вершины 10-этажного здание) короче, чем Галилей мог точно измерить с помощью доступные ему часы. Таким образом, прямой тест на постоянство Dv / Dt был невозможно для Галилео.

Какие из перечисленных уважительных причин, по которым Галилей не смог проверить? непосредственно, равна ли конечная скорость, достигаемая свободно падающим объектом пропорционально времени падения?
(а) Его определение было неверным.
б) Он не мог измерить скорость, достигаемую объектом непосредственно перед ним. ударился о землю.
в) не существовало приборов для измерения времени.
(г) Он не мог достаточно точно измерить обычные расстояния.
(e) Эксперименты в Италии запрещены.

В поисках логических следствий гипотезы Галилея


Неспособность Галилея провести прямые измерения для проверки своего гипотеза — что Dv / Dt постоянно в свободном падении — его не остановить.Он обратился к математике, чтобы вывести из этой гипотезы некоторые другие отношения, которые могут быть проверено измерениями с помощью имеющегося у него оборудования. Мы увидим что за несколько шагов он подошел гораздо ближе к кораблю отношений, который мог бы использовать чтобы проверить свою гипотезу.

Большие расстояния падения и большие интервалы времени падения составляют Конечно, легче измерить, чем малые значения Dd и Dt, что необходимо для поиска конечная скорость незадолго до удара падающего тела.Итак, Галилей попытался найти, рассуждая, как общее расстояние падения должно увеличиваться с увеличением общее время падения, если объекты падали с равномерным ускорением. Ты уже знаете, как найти общее расстояние от общего времени движения на постоянная скорость. Теперь мы выведем новое уравнение, которое связывает общую расстояние падения к общему времени падения для движения при постоянном ускорение. В этом мы не будем следовать собственным словам Галилея. вывод точно, но результаты будут такими же.Прежде всего напомним определение средней скорости как пройденное расстояние Dd, деленное на прошедшее время Dt:

v av = Dd / Dt
Это общее определение, которое можно использовать для вычисления среднего скорость от измерения Dd и Dt, независимо от того, Dd и Dt — большие или маленькие. Мы можем переписать уравнение как
Dd = v av x Dt
Это уравнение, по-прежнему являющееся определением v av , является всегда правда.Для особого случая движения с постоянной скоростью v, тогда v av = v а значит, Dd = v x Dt. Когда значение v известно (например, когда едет машина) с устойчивым показателем 60 миль / ч на спидометра), это уравнение можно использовать, чтобы определить, как далеко (Дд) машина поедет в любой момент времени интервал (Dt). Но в равномерно ускоренное движение скорость постоянно меняется — так какое значение мы можем использовать для v av ?

Ответ включает в себя немного алгебры и некоторые правдоподобные предположения.Галилей рассуждал (как и другие ранее), что для любого количество, которое изменяется равномерно, среднее значение только на полпути между начальным значением и конечным значением . Равномерно ускоренное движение, начиная с состояния покоя (где v начальное = 0 и окончание на скорости v final это правило говорит нам, что средняя скорость на полпути. В более общем плане средняя скорость будет между 0 и v final — то есть


v av = 1/2 v final .
(В более общем смысле средняя скорость будет
v av = (v начальная + v конечная ) / 2.
Если это рассуждение верно, то следует, что
Dd = 1/2 v final x Dt для равноускоренного движения из состояния покоя. Это отношение также не могли быть протестированы напрямую, потому что последний уравнение по-прежнему содержит коэффициент скорости. Что мы пытаемся приехать at — это уравнение, связывающее общее расстояние и общее время, без каких-либо нужно измерить скорость.
Теперь посмотрим на определение равномерного ускорения, данное Галилеем: а = Дв / Дт. Мы это соотношение можно переписать в виде
Dv = a x Dt. Значение Dv составляет всего v final — v начальный r и vinitial = 0 для движения, которое начинается из состояния покоя. Следовательно, мы можем написать
Dv = a x Dt
v final — v начальный = a x Dt
v финал = a x Dt
Теперь мы можем подставить это выражение для v final в уравнение для Dd выше.Таким образом, если движение начинается из состояния покоя, и если он равномерно ускоряется (и если среднее правило верно, как мы и предполагали) мы можем записать
Dd = 1/2 v final x Dt
= 1/2 (a x Dt) x Dt
Или перегруппировка условий.
Dd = 1/2 a (Dt) 2

Именно таких отношений искал Галилей — он связывает все расстояние Dd до полного времени Dt без учета скорости.

Но прежде чем закончить, мы упростим символы в уравнение, чтобы упростить его использование.Если мы измеряем расстояние и время от положение и момент начала движения (d начальный = 0 и t начальный = 0), то интервалы Dd и Dt имеют значения d final и т окончательный . Поскольку мы будем использовать выражение d final / t 2 final , во много раз проще написать в виде д / т 2

— подразумевается, что d и t означают общее расстояние и время интервал движения, начиная с покоя.Уравнение выше может поэтому можно записать более просто как

d окончательный = 1/2 a x t 2 окончательный
Помните, что это очень специализированное уравнение — оно дает общую расстояние падения как функция общего времени падения, но только если движение начинается из состояния покоя (v начальное = 0), если ускорение равномерно (a = константа), и если время и расстояние отсчитываются от начало (t начальное = 0 и d начальное = 0).

Галилей пришел к такому же выводу, хотя и не использовал алгебраические формы, чтобы выразить это. Поскольку мы имеем дело только с особая ситуация, в которой ускорение a постоянно, величина 2a также постоянна, и мы можем представить вывод в виде пропорция: при равномерном ускорении от состояния покоя пройденное расстояние равно пропорционально квадрату прошедшего времени, или

d окончательный / t 2 окончательный
Например, если равномерно ускоряющийся автомобиль, трогаясь с места, движется 10 м за первую секунду, в два раза больше времени он переместится в четыре раза, как далеко, или 40 м за первые две секунды.В первые 3 секунды он переместитесь в 9 раз дальше — или на 90 м. Другой способ выразить это отношение — сказать, что отношение d final to t 2 final имеет постоянное значение, то есть d конечный / t 2 конечный = постоянный. Таким образом логический результат первоначального предложения Галилея об определении униформы ускорение можно выразить следующим образом: если объект ускоряется равномерно из состояния покоя отношение d / t 2 должно быть постоянным.И наоборот, любое движение, для которого это отношение d и t 2 равно оказываются постоянными для разных расстояний и соответствующих им раз, мы вполне можем предположить, что это случай движения с равномерным ускорение по определению Галилео. Конечно, мы еще должны проверить гипотеза, которая свободно падающие тела действительно демонстрируют именно такое движение. Напомним, что ранее мы признались, что не можем напрямую проверить, имеет ли Dv / Dt постоянную стоимость. Галилей показал, что логическое следствие постоянного значения v / Dt будет постоянным соотношением d final к т 2 окончательный .Значения общего времени и расстояние падения было бы легче измерить, чем значения короткого интервалы Dd и Dt нужно было найти Дв. Однако измерение время падения все еще оставалось сложной задачей в Время Галилея. Итак, вместо прямой проверки своей гипотезы Галилей пошел еще дальше и вывел остроумный косвенный тест.

Почему уравнение d = 1/2 при 2 было более перспективным для Галилео, чем a = Dv / Dt в проверке своей гипотезы?

Если вы просто скомбинируете два уравнения Dd = v x Dt и Dv = a x Dt похоже, что можно было бы получить результат Dd = a xDt 2 .Что в этом плохого?

Понимая, что прямой количественный тест с быстрым и свободно падающее тело не было бы точным, Галилей предложил сделать испытание на объекте, который двигался менее быстро. Он предложил новый гипотеза:

если свободно падающее тело имеет ускорение, равное постоянно, то идеально круглый шар катится по идеально гладкой наклонная плоскость также будет иметь постоянное, хотя и меньшее, ускорение.

Таким образом, Галилей утверждал, что если d / t 2, постоянна для тело свободно падает из состояния покоя, это соотношение также будет постоянным, хотя и меньшего размера, для мяча, выпущенного из состояния покоя и качения расстояния по прямой наклонной плоскости.

Вот как Сальвиати описал собственное экспериментальное испытание Галилея в году. Новые науки :

Часть деревянного карниза или бруса, около 12 локтей. был взят длинный, шириной в пол локтя и толщиной в три пальца; на его край был прорезан каналом шириной чуть больше одного пальца; сделав эту канавку очень прямой, гладкой и отполированной, и выложили пергаментом, также как можно более гладким и отполированным, мы по нему катился твердый, гладкий и очень круглый бронзовый шар.Имея поместил эту доску в наклонное положение, приподняв за один конец какой-нибудь или на два локтя выше другого, мы катили мяч, как я только что сказал, вдоль канала, отмечая, как будет описано ниже, время, необходимое для спуска. Мы повторили этот эксперимент еще более одного раза, чтобы измерить время с такой точностью, чтобы отклонение между двумя наблюдениями никогда не превышало одной десятой пульса бить. Проделав эту операцию и убедившись в его надежность, теперь мы катили мяч только на четверть длины канала; и измерив время его спуска, мы нашли это ровно половина прежнего.Затем мы попробовали другие расстояния, сравнивая время для всей длины с временем для половины или с что за две трети или три четверти, или даже за любую фракцию; в таких экспериментов, повторенных сто раз, мы всегда обнаруживали, что пройденные пространства относились друг к другу как квадраты времен, и это было верно для всех наклонностей. . . канал, по которому мы катили мяч…
Обратите внимание на подробное описание экспериментальной установки.Сегодня ан экспериментатор добавил бы к словесному описанию любые подробные рисунки, схематические макеты или фотографии, необходимые для того, чтобы другие компетентные ученые продублируют эксперимент.


На этой картине, написанной в 1841 году Дж. Безцуоли, делается попытка воссоздать эксперимент, который Галилей якобы проделал, будучи лектором в Пизе. Слева и справа — люди злой воли: превозносить принца Джованни де Медичи (Галилей показал дноуглубительная машина, изобретенная князем для непригодности) и Галилея. научные оппоненты.Это были ведущие люди университетов; Они показаны здесь, склонившись над книгой Аристотеля, где она написана на черно-белые, что тела неравного веса падают с разными скорости. Галилей, самая высокая фигура слева от центра на картинке, в окружении группы студентов и последователей.

Угол наклона
Для каждого угла ускорение оказывается постоянным. Сферы катятся по плоскостям все более крутого наклона.В 90 ° наклонная плоскость соответствует свободному падению. (На самом деле мяч начнет скользить вместо того, чтобы катиться задолго до того, как угол стать таким большим.)

Свободное падение-Галилей описывает движение


Галилей вложил в эти строки много информации. Он достаточно ясно описывает свои процедуры и оборудование, чтобы позволить другим исследователи могут повторить эксперимент для себя, если захотят. Кроме того, он дает указание на возможность проведения последовательных измерений, и он повторяет два основных экспериментальных результата, которые, по его мнению, поддержать его гипотезу свободного падения.Давайте внимательно рассмотрим результаты.
(а) Во-первых, он обнаружил, что когда мяч скатывается по склону с фиксированной угол к горизонтали, отношение пройденного расстояния к квадрат соответствующего времени всегда был одинаковым. Например, если d 1 , d 2 и d 3 представляют собой расстояния, измеренные от одинаковая начальная точка на наклонной плоскости, и t 1 , t 2 , и t 3 — время, необходимое для скатывания этих расстояния, то d 1 / т 1 2 = d 2 / т 2 2 = d 3 / т 3 2 .

Как правило, для каждого угла наклона значение d / t 1 2 был постоянным. Галилей не полностью представил свои экспериментальные данные. деталь, которая с тех пор стала обычаем. Однако его эксперимент были повторены другими, и они получили результаты, которые параллельны его. Это эксперимент, который вы можете провести самостоятельно с помощью одного или двух других студентов.
(б) Второе экспериментальное открытие Галилея касается того, что происходит, когда изменен угол наклона плоскости.Он обнаружил, что всякий раз, когда угол менялся, отношение d / t 2 принимало новый значение, хотя для любого угла оно оставалось постоянным независимо от расстояние рулона. Галилей подтвердил это, повторив эксперимент «a. полных сто раз « для каждого из множества разных углов. После обнаружив, что отношение d / t 2 было постоянным для каждого угла наклон, для которого могут быть выполнены измерения t удобно, что Галилей был готов экстраполировать.Он пришел к выводу, что отношение d l / t 2 является постоянным даже для больших углы, при которых мяч движется слишком быстро для точного необходимо произвести измерения t. Наконец, Галилей пришел к выводу, что в частный случай, когда угол наклона стал 90 °, мяч будет двигаться прямо вниз — и так происходит в случае падающего объекта. По его рассуждениям, d / t 2 все еще будет некоторой константой в этом крайний случай (хотя он не мог сказать, какое числовое значение был.)

Поскольку Галилей пришел к выводу, что постоянное значение d / t

2 было характерно для равномерного ускорения, он мог, наконец, заключить, что свободное падение ускоренное движение. Теперь, когда вы знакомы с историческими концепциями свободного падения, переходим к эксперименту. Или вы можете посмотрите на таблицу некоторых актуальные данные о студентах. Данные были собраны и обработаны в соответствии с к описанному эксперименту.Это определенно демонстрирует, что существует постоянное ускорение. Хороший вопрос, который стоит задать студентам, — почему там это такая большая ошибка. Затем попросите учащихся изменить эксперимент. Когда все сказано и сделано, пройдите викторину.
Моя домашняя страница | Умная домашняя страница | Домашняя страница Улыбки | Эксперимент | Анализ | Контрольный опрос

Движение в двух измерениях | Безграничная физика

Постоянная скорость

Объект, движущийся с постоянной скоростью, должен иметь постоянную скорость в постоянном направлении.

Цели обучения

Изучите термины для постоянной скорости и их применимость к ускорению

Основные выводы

Ключевые моменты
  • Постоянная скорость означает, что движущийся объект движется по прямой с постоянной скоростью.
  • Эта строка может быть представлена ​​алгебраически как: [latex] \ text {x} = \ text {x} _0 + \ text {vt} [/ latex], где [latex] \ text {x} _0 [/ latex] представляет положение объекта в [latex] \ text {t} = 0 [/ latex], а наклон линии указывает скорость объекта.
  • Скорость может быть положительной или отрицательной и указывается знаком нашего наклона. Это говорит нам, в каком направлении движется объект.
Ключевые термины
  • постоянная скорость : Движение, которое не меняется ни по скорости, ни по направлению.

Движение с постоянной скоростью — одна из простейших форм движения. Этот тип движения возникает, когда объект движется (или скользит) в присутствии небольшого или незначительного трения, подобно тому, как хоккейная шайба скользит по льду.Чтобы иметь постоянную скорость, объект должен иметь постоянную скорость в постоянном направлении. Постоянное направление заставляет объект двигаться по прямой траектории.

Второй закон Ньютона ([latex] \ text {F} = \ text {ma} [/ latex]) предполагает, что когда к объекту прикладывается сила, объект будет испытывать ускорение. Если ускорение равно 0, объект не должен подвергаться никаким внешним силам. Математически это можно представить следующим образом:

[латекс] \ text {a} = \ frac {\ text {dv}} {\ text {dt}} = 0 ~ \ Rightarrow ~ \ text {v} = \ text {const} [/ latex].

Если объект движется с постоянной скоростью, график зависимости расстояния от времени ([latex] \ text {x} [/ latex] vs. [latex] \ text {t} [/ latex]) показывает такое же изменение положение по каждому интервалу времени. Следовательно, движение объекта с постоянной скоростью представлено прямой линией: [latex] \ text {x} = \ text {x} _0 + \ text {vt} [/ latex], где [latex] \ text {x} _0 [/ latex] — это смещение, когда [latex] \ text {t} = 0 [/ latex] (или в точке пересечения оси Y).

Движение с постоянной скоростью : Когда объект движется с постоянной скоростью, он не меняет ни направления, ни скорости, и поэтому отображается как прямая линия на графике как расстояние во времени.

Вы также можете получить скорость объекта, если знаете его след во времени. Имея график, как в, мы можем вычислить скорость по изменению расстояния с течением времени. Графически скорость можно интерпретировать как наклон линии. Скорость может быть положительной или отрицательной и указывается знаком нашего наклона. Это говорит нам, в каком направлении движется объект.

Постоянное ускорение

Анализ двумерного движения снаряда выполняется путем разбиения его на два движения: по горизонтальной и вертикальной осям.

Цели обучения

Анализировать двумерное движение снаряда по горизонтальной и вертикальной осям

Основные выводы

Ключевые моменты
  • Постоянное ускорение при движении в двух измерениях обычно происходит по образцу снаряда.
  • Движение снаряда — это движение объекта, брошенного или выброшенного в воздух, с учетом только (вертикального) ускорения силы тяжести.
  • Мы анализируем двумерное движение снаряда, разбивая его на два независимых одномерных движения по вертикальной и горизонтальной осям.
Ключевые термины
  • кинематика : или относящаяся к движению или кинематике

Движение снаряда — это движение объекта, брошенного или выброшенного в воздух, подверженное только силе тяжести. Объект называется снарядом, а его путь называется его траекторией. Движение падающих предметов — это простой одномерный тип движения снаряда, при котором нет горизонтального движения. В двумерном движении снаряда, например футбольного мяча или другого брошенного объекта, есть как вертикальная, так и горизонтальная составляющие движения.

Движение снаряда : Бросок камня или удар ногой по мячу, как правило, создает образец движения снаряда, который имеет как вертикальную, так и горизонтальную составляющие.

Самый важный факт, о котором следует помнить, это то, что движения по перпендикулярным осям независимы и поэтому могут быть проанализированы отдельно. Ключ к анализу двумерного движения снаряда состоит в том, чтобы разбить его на два движения, одно по горизонтальной оси, а другое — по вертикали. Чтобы описать движение, мы должны иметь дело со скоростью и ускорением, а также со смещением.2} [/ latex] (мы предполагаем, что движение происходит на достаточно малых высотах у поверхности земли, чтобы ускорение свободного падения было постоянным). 2 + 2 \ text {a} _ \ text {y} (\ text {y} — \ text {y} _0) [/ latex]

Мы анализируем двумерное движение снаряда, разбивая его на два независимых одномерных движения по вертикальной и горизонтальной осям.Горизонтальное движение простое, потому что [latex] \ text {a} _ \ text {x} = 0 [/ latex] и [latex] \ text {v} _ \ text {x} [/ latex], таким образом, является постоянным. Скорость в вертикальном направлении начинает уменьшаться по мере подъема объекта; в самой высокой точке вертикальная скорость равна нулю. Когда объект снова падает на Землю, вертикальная скорость снова увеличивается по величине, но указывает в направлении, противоположном начальной вертикальной скорости. Движения [latex] \ text {x} [/ latex] и [latex] \ text {y} [/ latex] могут быть рекомбинированы для получения общей скорости в любой заданной точке траектории.

3.6 Определение скорости и смещения по ускорению — University Physics Volume 1

3 Движение по прямой

Цели обучения

К концу этого раздела вы сможете:

  • Выведите кинематические уравнения для постоянного ускорения с помощью интегрального исчисления.
  • Используйте интегральную формулировку кинематических уравнений при анализе движения.
  • Найдите функциональную форму зависимости скорости от времени с учетом функции ускорения.
  • Найдите функциональную форму зависимости положения от времени с учетом функции скорости.

В этом разделе предполагается, что у вас достаточно знаний в области вычислений, чтобы быть знакомыми с интеграцией. В разделах «Мгновенная скорость и скорость», «Среднее и мгновенное ускорение» мы ввели кинематические функции скорости и ускорения с использованием производной. Взяв производную функции положения, мы нашли функцию скорости, и аналогичным образом взяв производную функции скорости, мы нашли функцию ускорения.Используя интегральное исчисление, мы можем работать в обратном направлении и вычислять функцию скорости из функции ускорения и функцию положения из функции скорости.

Кинематические уравнения из интегрального исчисления

Начнем с частицы с ускорением a (t) — известная функция времени. Поскольку производной функции скорости по времени является ускорение,

мы можем взять неопределенный интеграл от обеих частей, найдя

, где C 1 — постоянная интегрирования.С

, скорость определяется как

Аналогично, производная по времени функции положения является функцией скорости,

Таким образом, мы можем использовать те же математические манипуляции, которые мы только что использовали, и найти

, где C 2 — вторая постоянная интегрирования.

Используя эти интегралы, мы можем вывести кинематические уравнения для постоянного ускорения.Имея a ( t ) = a константу, и выполняя интегрирование в (рисунок), мы находим

Если начальная скорость v (0) = v 0 , то

Тогда C 1 = v 0 и

(Уравнение). Подстановка этого выражения в (рисунок) дает

Выполняя интеграцию, находим

Если x (0) = x 0 , имеем

так, C 2 = x 0 .Подставляя обратно в уравнение для x ( t ), мы, наконец, имеем

(Уравнение).

Пример

Движение моторной лодки

Моторная лодка движется с постоянной скоростью 5,0 м / с, когда начинает замедляться, чтобы прибыть в док. Его ускорение

. а) Какова функция скорости моторной лодки? (б) В какое время скорость достигает нуля? (c) Какова функция местоположения моторной лодки? (d) Каково смещение моторной лодки с момента начала замедления до момента, когда скорость равна нулю? (e) Постройте график функций скорости и положения.

Стратегия

(a) Чтобы получить функцию скорости, мы должны интегрировать и использовать начальные условия, чтобы найти постоянную интегрирования. (b) Мы устанавливаем функцию скорости равной нулю и решаем для t . (c) Аналогично, мы должны интегрировать, чтобы найти функцию положения, и использовать начальные условия, чтобы найти постоянную интегрирования. (d) Поскольку начальное положение принимается равным нулю, нам нужно только оценить функцию положения на

.

.

Решение

Возьмем t = 0 за время начала замедления лодки.

  1. Из функциональной формы ускорения мы можем решить (рисунок), чтобы получить v ( t ): [раскрыть-ответ q = ”136447 ″] Показать ответ [/ раскрыть-ответ]
    [hidden-answer a = ”136447 ″]

    При t = 0 имеем v (0) = 5,0 м / с = 0 + C1, поэтому C1 = 5,0 м / с или

    . [/ Hidden-answer]

  2. [show-answer q = ”967265 ″] Показать ответ [/ show-answer]
    [hidden-answer a =” 967265 ″]

    [/ hidden-answer]

  3. Решить (рисунок): [show-answer q = ”251505 ″] Показать ответ [/ show-answer]
    [hidden-answer a = ”251505 ″]

    При t = 0 мы устанавливаем x (0) = 0 = x0, поскольку нас интересует только смещение с момента начала замедления лодки.У нас

    Следовательно, уравнение для позиции —

    [/ hidden-answer]

  4. [show-answer q = ”330950 ″] Показать ответ [/ show-answer]
    [hidden-answer a =” 330950 ″] Поскольку начальная позиция принимается равной нулю, нам нужно вычислить x (t) только тогда, когда скорость равна нулю. Это происходит при t = 6,3 с. Следовательно, смещение равно

    [/ hidden-answer]

Рис. 3.30 (a) Скорость катера как функция времени.Катер снижает скорость до нуля за 6,3 с. Иногда скорость становится отрицательной — это означает, что лодка меняет направление. (b) Положение моторной лодки как функция времени. В момент времени t = 6,3 с скорость равна нулю, и лодка остановилась. В разы больше, чем это значение, скорость становится отрицательной — это означает, что если лодка продолжает двигаться с тем же ускорением, она меняет направление на противоположное и направляется обратно к месту своего зарождения.
Значение

Функция ускорения линейна по времени, поэтому интегрирование включает простые полиномы.На (Рисунок) мы видим, что если мы продолжим решение за точку, когда скорость равна нулю, скорость станет отрицательной, и лодка изменит направление на противоположное. Это говорит нам о том, что решения могут предоставить нам информацию, выходящую за рамки наших непосредственных интересов, и мы должны быть осторожны при их интерпретации.

Проверьте свое понимание

Частица стартует из состояния покоя и имеет функцию ускорения

. а) Что такое функция скорости? б) Что такое функция положения? (c) Когда скорость равна нулю?

[показывать-ответ q = ”fs-id1168057352922 ″] Показать решение [/ показывать-ответ]

[скрытый-ответ a = ”fs-id1168057352922 ″]

  1. Функция скорости представляет собой интеграл от функции ускорения плюс постоянную интегрирования.По (Рисунок),

    Поскольку v (0) = 0, имеем C 1 = 0; итак,

  2. По (рисунок),

    . Так как x (0) = 0, мы имеем C 2 = 0 и

  3. Скорость может быть записана как v ( t ) = 5 t (1 — t ), что равно нулю при t = 0 и t = 1 с.

[/ hidden-answer]

Сводка

  • Интегральное исчисление дает нам более полную формулировку кинематики.
  • Если известно ускорение a ( t ), мы можем использовать интегральное исчисление для получения выражений для скорости v ( t ) и положения x ( t ).
  • Если ускорение постоянное, интегральные уравнения сводятся к (Рисунок) и (Рисунок) для движения с постоянным ускорением.

Ключевые уравнения

Рабочий объем

Полный рабочий объем

Средняя скорость

Мгновенная скорость

Средняя скорость

Мгновенная скорость

Среднее ускорение

Мгновенное ускорение

Положение от средней скорости

Средняя скорость

Скорость от разгона

Положение от скорости и ускорения

Скорость на расстоянии

Скорость свободного падения

Высота свободного падения

Скорость свободного падения с высоты

Скорость от разгона

Положение от скорости

Концептуальные вопросы

Если задана функция ускорения, какая дополнительная информация необходима для нахождения функции скорости и функции положения?

Проблемы

Ускорение частицы меняется со временем в соответствии с уравнением

.Изначально скорость и положение равны нулю. а) Какова скорость как функция времени? б) Какое положение зависит от времени?

Между t = 0 и t = t 0 ракета движется прямо вверх с ускорением, задаваемым

.

, где A и B — константы. (a) Если x в метрах, а t в секундах, каковы единицы измерения A и B ? (b) Если ракета стартует из состояния покоя, как изменяется скорость от t = 0 до t = t 0 ? (c) Если ее начальное положение равно нулю, каково положение ракеты в зависимости от времени в течение этого же временного интервала?

[show-answer q = ”fs-id1168055134758 ″] Показать решение [/ show-answer]

[скрытый-ответ a = ”fs-id1168055134758 ″]

а.

;
г.

;

г.

[/ hidden-answer]

Скорость частицы, движущейся вдоль оси x- , изменяется со временем в соответствии с

.

, где A = 2 м / с, B = 0,25 м и

. Определите ускорение и положение частицы при t = 2,0 с и t = 5.0 с. Предположим, что

.

Покоящаяся частица покидает начало координат со скоростью, увеличивающейся со временем согласно v ( t ) = 3,2 t м / с. На 5,0 с скорость частицы начинает уменьшаться в соответствии с [16,0 — 1,5 ( т — 5,0)] м / с. Это уменьшение продолжается до t = 11,0 с, после чего скорость частицы остается постоянной и составляет 7,0 м / с. а) Каково ускорение частицы как функция времени? (б) Каково положение частицы при t = 2.0 с, т = 7,0 с и т = 12,0 с?

[показывать-ответ q = ”fs-id1168055121296 ″] Показать решение [/ показывать-ответ]

[скрытый-ответ a = ”fs-id1168055121296 ″]

а.

;
г.

[/ hidden-answer]

Дополнительные проблемы

Профессиональный бейсболист Нолан Райан мог подавать бейсбольный мяч со скоростью примерно 160,0 км / ч. При такой средней скорости, сколько времени потребовалось мячу, брошенному Райаном, чтобы достичь своей тарелки, а это 18.4 м от насыпи питчера? Сравните это со средним временем реакции человека на визуальный стимул, которое составляет 0,25 с.

Самолет вылетает из Чикаго и совершает 3000-километровый перелет в Лос-Анджелес за 5,0 ч. Второй самолет вылетает из Чикаго через полчаса и одновременно прибывает в Лос-Анджелес. Сравните средние скорости двух плоскостей. Не обращайте внимания на кривизну Земли и разницу в высоте между двумя городами.

[show-answer q = ”fs-id1168055151090 ″] Показать решение [/ show-answer]

[скрытый-ответ a = ”fs-id1168055151090 ″]

Двигайтесь на запад в положительном направлении.

1-й самолет:

2-й самолет

[/ hidden-answer]

Необоснованные результаты Велосипедист едет на 16,0 км на восток, затем на 8,0 км на запад, затем на 8,0 км на восток, затем на 32,0 км на запад и, наконец, на 11,2 км на восток. Если его средняя скорость составляет 24 км / ч, сколько времени ему потребовалось, чтобы завершить поездку? Это разумное время?

У объекта ускорение

.

. На

, его скорость

.Определите скорости объекта на

и

.

[показывать-ответ q = ”fs-id1168055302745 ″] Показать решение [/ показывать-ответ]

[скрытый-ответ a = ”fs-id1168055302745 ″]

,

;

[/ hidden-answer]

Частица движется по оси x согласно уравнению

г.Какие скорость и ускорение у

с и

с?

Частица, движущаяся с постоянным ускорением, имеет скорость

.

в

с и

в

с. Что такое ускорение частицы?

[показывать-ответ q = ”fs-id1168055307822 ″] Показать решение [/ показывать-ответ]

[скрытый-ответ a = ”fs-id1168055307822 ″]

[/ hidden-answer]

Поезд движется по крутому склону с постоянной скоростью (см. Следующий рисунок), когда его камбуз отрывается и начинает свободно катиться по рельсам.Через 5,0 с камбуз отстает от поезда на 30 м. Какое ускорение у камбуза?

Электрон движется по прямой со скоростью

м / с. Он входит в область длиной 5,0 см, где испытывает ускорение

по той же прямой. а) Какова скорость электрона, когда он выходит из этой области? б) Сколько времени требуется электрону, чтобы пересечь область?

[показывать-ответ q = ”fs-id1168055302554 ″] Показать решение [/ показывать-ответ]

[скрытый-ответ a = ”fs-id1168055302554 ″]

а.

;

г.

[/ hidden-answer]

Водитель «скорой помощи» доставляет пациента в больницу. На скорости 72 км / ч она замечает, что светофор на ближайших перекрестках стал желтым. Чтобы добраться до перекрестка до того, как загорится красный свет, она должна проехать 50 м за 2,0 с. (a) Какое минимальное ускорение должно быть у машины скорой помощи, чтобы добраться до перекрестка, прежде чем загорится красный свет? б) Какова скорость машины скорой помощи, когда она подъезжает к перекрестку?

Мотоцикл, который замедляет скорость, равномерно покрывает 2.0 последовательных км за 80 с и 120 с соответственно. Рассчитайте (а) ускорение мотоцикла и (б) его скорость в начале и в конце 2-километровой поездки.

[показывать-ответ q = ”fs-id1168057524743 ″] Показать решение [/ показывать-ответ]

[скрытый-ответ a = ”fs-id1168057524743 ″]

;

решайте одновременно, чтобы получить

и

, что составляет

.Скорость в конце рейса

.
[/ hidden-answer]

Велосипедист едет из пункта А в пункт Б за 10 мин. В течение первых 2,0 минут поездки она поддерживает равномерное ускорение

.

. Затем она движется с постоянной скоростью следующие 5,0 мин. Затем она замедляется с постоянной скоростью, так что она приходит в состояние покоя в точке B на 3,0 мин позже. (а) Нарисуйте график зависимости скорости от времени для поездки. (б) Какое ускорение произошло за последние 3 минуты? (c) Как далеко едет велосипедист?

Два поезда движутся со скоростью 30 м / с в противоположных направлениях по одному и тому же пути.Инженеры одновременно видят, что они идут на встречу, и включают тормоза, когда они находятся на расстоянии 1000 м друг от друга. Предполагая, что оба поезда имеют одинаковое ускорение, каким должно быть это ускорение, если поезда должны останавливаться незадолго до столкновения?

[show-answer q = ”fs-id1168055171872 ″] Показать решение [/ show-answer]

[скрытый-ответ a = ”fs-id1168055171872 ″]

[/ hidden-answer]

Грузовик длиной 10,0 м, движущийся с постоянной скоростью 97.0 км / ч проезжает автомобиль длиной 3,0 м, движущийся с постоянной скоростью 80,0 км / ч. Сколько времени проходит между моментом, когда передняя часть грузовика сравняется с задней частью автомобиля, и моментом, когда задняя часть грузовика сравняется с передней частью автомобиля?

Полицейская машина ждет в укрытии немного в стороне от шоссе. Полицейская машина обнаруживает мчащуюся машину со скоростью 40 м / с. В тот момент, когда машина, превышающая скорость, проезжает мимо полицейской машины, полицейская машина ускоряется из состояния покоя со скоростью 4 м / с 2 , чтобы поймать ускоряющуюся машину.Сколько времени нужно полицейской машине, чтобы догнать мчащуюся машину?

[показывать-ответ q = ”fs-id1168055306834 ″] Показать решение [/ показывать-ответ]

[скрытый-ответ a = ”fs-id1168055306834 ″]

Уравнение для ускоряющегося автомобиля: этот автомобиль имеет постоянную скорость, которая является средней скоростью, и не ускоряется, поэтому используйте уравнение для перемещения с

.

:

; Уравнение для полицейской машины: эта машина ускоряется, поэтому используйте уравнение для перемещения с

.

и

, так как патрульная машина трогается с места:

; Теперь у нас есть уравнение движения для каждой машины с общим параметром, который можно исключить, чтобы найти решение.В этом случае мы решаем

. Шаг 1, исключая

:

; Шаг 2, решение для

:

. Мчащийся автомобиль имеет постоянную скорость 40 м / с, которая является его средней скоростью. Ускорение полицейской машины составляет 4 м / с 2 . Оценивая t , время, за которое полицейская машина достигает скорости, мы получаем

.
[/ hidden-answer]

Пабло бежит полумарафон со скоростью 3 м / с. Другой бегун, Джейкоб, с той же скоростью отстает от Пабло на 50 метров. Джейкоб начинает ускоряться со скоростью 0,05 м / с 2 . а) Сколько времени нужно Иакову, чтобы поймать Пабло? б) Какое расстояние преодолел Иаков? в) Какова конечная скорость Иакова?

Необоснованные результаты Бегун приближается к финишу и находится на расстоянии 75 м; ее средняя скорость в этом положении составляет 8 м / с.В этот момент она замедляется со скоростью 0,5 м / с 2 . Сколько времени ей нужно, чтобы пересечь финишную черту с расстояния 75 м? Это разумно?

[показывать-ответ q = ”fs-id1168055381859 ″] Показать решение [/ показывать-ответ]

[скрытый-ответ a = ”fs-id1168055381859 ″]

На этом ускорении она доходит до полной остановки на

.

, но пройденное расстояние —

, что меньше расстояния, на которое она отошла от финиша, поэтому она никогда не финиширует.
[/ hidden-answer]

Самолет ускоряется со скоростью 5,0 м / с 2 за 30,0 с. За это время он преодолевает расстояние 10,0 км. Каковы начальная и конечная скорости самолета?

Сравните расстояние, пройденное объектом, скорость которого в два раза превышает начальную скорость, с объектом, который изменяет свою скорость в четыре раза по сравнению с начальной скоростью за тот же период времени. Ускорения обоих объектов постоянны.

[показывать-ответ q = ”fs-id1168055323241 ″] Показать решение [/ показывать-ответ]

[скрытый-ответ a = ”fs-id1168055323241 ″]

[/ hidden-answer]

Объект движется на восток с постоянной скоростью и находится в позиции

.

.(а) С каким ускорением должен иметь объект, чтобы его полное смещение в более позднее время стало равным нулю t ? (б) Какова физическая интерпретация решения в случае

?

?

Мяч бросается прямо вверх. На своем пути вверх он проходит окно высотой 2,00 м над землей на высоте 7,50 м и проходит за 1,30 с. Какая была начальная скорость мяча?

[показывать-ответ q = ”fs-id11680553

″] Показать решение [/ показывать-ответ]

[скрытый-ответ a = ”fs-id11680553

″]

скорость внизу окна.

[/ hidden-answer]

Монета сбрасывается с воздушного шара, который находится на высоте 300 м над землей и поднимается вверх со скоростью 10,0 м / с. Для монеты найдите (а) максимальную достигнутую высоту, (б) ее положение и скорость через 4,00 с после того, как она была выпущена, и (в) время до того, как она упадет на землю.

Мягкий теннисный мяч падает на твердый пол с высоты 1,50 м и отскакивает на высоту 1,10 м. (а) Рассчитайте его скорость непосредственно перед тем, как он ударится об пол.(б) Рассчитайте его скорость сразу после того, как он покинет пол на обратном пути вверх. (c) Рассчитайте его ускорение во время контакта с полом, если этот контакт длится 3,50 мс

(d) Насколько сильно мяч сжался во время столкновения с полом, если предположить, что пол абсолютно жесткий?

[показывать-ответ q = ”fs-id1168055325521 ″] Показать решение [/ показывать-ответ]

[скрытый-ответ a = ”fs-id1168055325521 ″]

а.

;
г.

;

г.

;

г.

[/ hidden-answer]

Необоснованные результаты . Капля дождя падает из облака на высоте 100 м над землей. Пренебрегайте сопротивлением воздуха. Какова скорость капли дождя, когда она падает на землю? Это разумное число?

Сравните время в воздухе баскетболиста, который прыгает на 1,0 м вертикально от пола, с временем игрока, прыгнувшего 0.3 м по вертикали.

[показывать-ответ q = ”fs-id1168057418927 ″] Показать решение [/ показывать-ответ]

[скрытый-ответ a = ”fs-id1168057418927 ″]

Рассмотрим падение игроков с высоты 1,0 м и 0,3 м.

0,9 с

0,5 с

[/ hidden-answer]

Предположим, что человеку требуется 0,5 с, чтобы отреагировать и переместить руку, чтобы поймать предмет, который он уронил. (а) Как далеко объект падает на Землю, где

(b) Как далеко объект падает на Луну, где ускорение свободного падения составляет 1/6 от земного?

Воздушный шар поднимается с уровня земли с постоянной скоростью 3.0 м / с. Через минуту после взлета с воздушного шара случайно падает мешок с песком. Вычислите (а) время, необходимое мешку с песком, чтобы достичь земли и (б) скорость мешка с песком, когда он ударяется о землю.

[показывать-ответ q = ”fs-id1168055469821 ″] Показать решение [/ показывать-ответ]

[скрытый-ответ a = ”fs-id1168055469821 ″]

а.

с положительным корнем;
г.

[/ hidden-answer]

(a) На Олимпийских играх 2008 года в Пекине Усэйн Болт из Ямайки установил мировой рекорд в беге на 100 метров среди мужчин.Болт «прошел» по финишу со временем 9,69 с. Если мы предположим, что Болт ускорялся в течение 3,00 с, чтобы достичь своей максимальной скорости, и сохранял эту скорость до конца гонки, вычислите его максимальную скорость и его ускорение. (b) Во время той же Олимпиады Болт также установил мировой рекорд в беге на 200 м со временем 19,30 с. Если исходить из тех же предположений, что и для бега на 100 м, какова была его максимальная скорость в этой гонке?

Предмет падает с высоты 75,0 м над уровнем земли.(а) Определите расстояние, пройденное за первую секунду. (b) Определите конечную скорость, с которой объект ударяется о землю. (c) Определите расстояние, пройденное за последнюю секунду движения до удара о землю.

[show-answer q = ”fs-id1168055273683 ″] Показать решение [/ show-answer]

[скрытый-ответ a = ”fs-id1168055273683 ″]

а.

;
г.

;

г.

[/ hidden-answer]

Стальной шар падает на твердый пол с высоты 1.50 м и подборы на высоту 1,45 м. (а) Рассчитайте его скорость непосредственно перед тем, как он ударится об пол. (б) Рассчитайте его скорость сразу после того, как он покинет пол на обратном пути вверх. (c) Рассчитайте его ускорение при контакте с полом, если этот контакт длится 0,0800 мс

(d) Насколько сильно мяч сжался во время столкновения с полом, если предположить, что пол абсолютно жесткий?

Объект упал с крыши здания высотой h .За последнюю секунду спуска он падает на расстояние ч /3. Рассчитайте высоту здания.

[show-answer q = ”fs-id11680554 ″] Показать решение [/ show-answer]

[скрытый-ответ a = ”fs-id11680554 ″]

, ч = общая высота и время падения на землю

за т — за 1 секунду падает 2/3 ч

или

т = 5.45 с и ч = 145,5 м. Другой корень меньше 1 с. Проверить т = 4,45 с

м

[/ hidden-answer]

Задачи

В беге на 100 м победитель определяется за 11,2 с. Время занявшего второе место — 11,6 с. Как далеко игрок, занявший второе место, отстает от победителя, когда она пересекает финишную черту? Предположим, что скорость каждого бегуна постоянна на протяжении всего забега.

Положение частицы, движущейся по оси x , изменяется со временем в соответствии с

.

г.Найдите (a) скорость и ускорение частицы как функции времени, (b) скорость и ускорение при t = 2,0 с, (c) время, в которое положение является максимальным, (d) время при скорость которого равна нулю, и (e) максимальное положение.

[показывать-ответ q = ”fs-id1168055269782 ″] Показать решение [/ показывать-ответ]

[скрытый-ответ a = ”fs-id1168055269782 ″]

а.

;
г.

; c.Наклон функции положения равен нулю или скорость равна нулю. Есть два возможных решения: t = 0, что дает x = 0, или t = 10,0 / 12,0 = 0,83 с, что дает x = 1,16 м. Второй ответ — правильный выбор; d. 0,83 с (э) 1,16 м

[/ hidden-answer]

Велосипедист мчится в конце гонки, чтобы одержать победу. Она имеет начальную скорость 11,5 м / с и ускоряется со скоростью 0,500 м / с 2 за 7.00 с. а) Какова ее конечная скорость? (b) Велосипедист продолжает движение на этой скорости до финиша. Если она находится в 300 м от финиша, когда начинает ускоряться, сколько времени она сэкономила? (c) Победитель, занявший второе место, был на 5,00 м впереди, когда победитель начал ускоряться, но он не смог ускориться и ехал со скоростью 11,8 м / с до финиша. Какая разница во времени финиша в секундах между победителем и занявшим второе место? Как далеко назад был занявший второе место, когда победитель пересек финишную черту?

В 1967 году новозеландец Берт Манро установил мировой рекорд для индийского мотоцикла на соляных равнинах Бонневиль в штате Юта — 295 человек.38 км / ч. Трасса в одну сторону была протяженностью 8,00 км. Скорость ускорения часто описывается временем, необходимое для достижения 96,0 км / ч из состояния покоя. Если на этот раз было 4,00 с, и Берт ускорялся с этой скоростью, пока не достиг максимальной скорости, сколько времени потребовалось Берту, чтобы пройти курс?

[показывать-ответ q = ”fs-id1168057239219 ″] Показать решение [/ показывать-ответ]

[скрытый-ответ a = ”fs-id1168057239219 ″]

, 295,38 км / ч = 82,05 м / с,

время разгона до максимума

пройденное расстояние при разгоне

при постоянной скорости

, так что общее время

.

[/ hidden-answer]

.

Добавить комментарий

Ваш адрес email не будет опубликован. Обязательные поля помечены *